GI Exam 5

अब Quizwiz के साथ अपने होमवर्क और परीक्षाओं को एस करें!

The nurse is developing a plan of care for a client with cirrhosis and ascites. Which nursing actions should be included in the care plan for this client? Select all that apply. 1. Monitor daily weight. 2. Measure abdominal girth. 3. Monitor respiratory status. 4. Place the client in a supine position. 5. Assist the client with care as needed.

1. Monitor daily weight. 2. Measure abdominal girth. 3. Monitor respiratory status. 5. Assist the client with care as needed. Rationale: Ascites is a problem because as more fluid is retained, it pushes up on the diaphragm, thereby impairing the client's breathing patterns. The client should be placed in a semi-Fowler's position with the arms supported on a pillow to allow for free diaphragm movement. The correct options identify appropriate nursing interventions to be included in the plan of care for the client with ascites.

A client is diagnosed with a gastrointestinal (GI) bleed, and the bleeding has been controlled. Antacids are prescribed to be administered every hour. The nurse administers the antacids and should plan to maintain an approximate gastric pH of which value? 1. 3 2. 6 3. 9 4. 15

2. 6 Rationale: During the first few days after hemorrhage, gastric pH should be increased to between 5.5 and 7.0 and maintained at this level to control secretory activity. Ranitidine (Zantac) or cimetidine (Tagamet) may be prescribed in addition to antacids to accomplish this. The use of antacids complements the effectiveness of histamine 2 receptor antagonists for maintaining the pH level of gastric secretions. Option 1 identifies an acidic pH; options 3 and 4 identify a pH that are alkaline.

A client with ulcerative colitis has a prescription to begin a salicylate medication to reduce inflammation. What instruction should the nurse give the client regarding when to take this medication? 1. On arising 2. After meals 3. On an empty stomach 4. 30 minutes before meals

2. After meals Rationale: Salicylate compounds such as sulfasalazine (Azulfidine) act by inhibiting prostaglandin synthesis and reducing inflammation. The nurse teaches the client to take the medication with a full glass of water and increase fluid intake throughout the day. The medication needs to be taken after meals to reduce gastrointestinal irritation. The other options are incorrect and could cause gastric irritation.

Discharge teaching for a client with chronic pancreatitis should include which instructions? 1. Alcohol should be consumed in moderation. 2. Avoid caffeine, because it may aggravate symptoms. 3. Diet should be high in carbohydrates, fats, and proteins. 4. Frothy fatty stools indicate that enzyme replacement is working.

2. Avoid caffeine, because it may aggravate symptoms. Rationale: Knowing that caffeinated beverages, such as coffee, tea, and soda, will worsen symptoms, such as pain, will direct you to select the correct option. Alcohol can precipitate an attack of pancreatitis and needs to be avoided. The recommended diet is moderate carbohydrates, low fat, and moderate protein. Frothy fatty stools indicate that the replacement enzyme dose needs to be increased. IGGY

The client with gastroesophageal reflux disease (GERD) has a new prescription for pantoprazole (Protonix). Which instruction should the nurse provide to the client? 1. Chew the pill thoroughly. 2. Swallow the tablet whole. 3. Headache is expected to occur. 4. Crush the pill if it is difficult to swallow.

2. Swallow the tablet whole. Rationale: Protonix, a proton pump inhibitor, is a delayed-release medication and should be swallowed whole. It should not be chewed or crushed. Headache is a potential side effect of the medication and should be reported to the health care provider if it is troublesome.

A client is resuming a diet after hemigastrectomy and the nurse provides dietary instructions. Which statement by the client indicates a need for further teaching? 1. "I plan to lie down after eating." 2. "I will eat six small meals per day." 3. "I will drink plenty of liquids with meals." 4. "I know to exclude concentrated sweets in my diet."

3. "I will drink plenty of liquids with meals." Rationale: The client who has had a hemigastrectomy is at risk for dumping syndrome. This client should be placed on a diet that is high in protein, moderate in fat, and high in calories. The client should avoid drinking liquids with meals. Frequent small meals are encouraged, and the client should avoid concentrated sweets. IGGY

The nurse has taught the client about an upcoming endoscopic retrograde cholangiopancreatography procedure. The nurse determines that the client needs further information if the client makes which statement? 1. "I know I must sign the consent form." 2. "I hope the throat spray keeps me from gagging." 3. "I'm glad I don't have to lie still for this procedure." 4. "I'm glad some IV medication will be given to relax me."

3. "I'm glad I don't have to lie still for this procedure." Rationale: The client does have to lie still for endoscopic retrograde cholangiopancreatography (ERCP), which takes about 1 hour to perform. The client also has to sign a consent form. Intravenous sedation is given to relax the client, and an anesthetic spray is used to help keep the client from gagging as the endoscope is passed. IGGY & Pagana Pagana

The adult client with hepatic encephalopathy has a serum ammonia level of 95 mcg/dL and receives treatment with lactulose (Chronulac). The nurse determines that the client had the best and most realistic response if the serum ammonia level changed to which value after medication administration? 1. 5 mcg/dL 2. 10 mcg/dL 3. 40 mcg/dL 4. 90 mcg/dL

3. 40 mcg/dL Rationale: The normal serum ammonia level is 10 to 80 mcg/dL. In the client with hepatic encephalopathy, the serum level is not likely to drop below normal, nor is it likely to drop into the low-normal range. The most optimal but realistic change would be to 40 mcg/dL, which falls into the middle-normal range. A level of 90 mcg/dL represents insufficient effect of the medication. The nurse should also monitor the client for signs and symptoms that indicate improvement in the condition. IGGY & Pagana Pagana

Vasopressin therapy is prescribed for a client with a diagnosis of bleeding esophageal varices. The nurse is preparing to administer the medication to the client. Which essential item is needed during the administration of this medication? 1. An airway 2. A suction setup 3. A cardiac monitor 4. A tracheotomy set

3. A cardiac monitor Rationale: The major action of vasopressin is constriction of the splanchnic blood flow. Continuous electrocardiogram and blood pressure monitoring are essential because of the constrictive effects of the medication on the coronary arteries. Options 1, 2, and 4 are not essential items required during the administration of this medication. However, these items may be needed if a complication arises.

A nurse manager is providing an educational session to nursing staff members about the phases of viral hepatitis. The nurse manager tells the staff that which clinical manifestation(s) is/are primarily characteristic of the preicteric phase? 1. Pruritus 2. Right upper quadrant pain 3. Fatigue, anorexia, and nausea 4. Jaundice, dark-colored urine, and clay-colored stools

3. Fatigue, anorexia, and nausea Rationale: In the preicteric phase the client has nonspecific complaints of fatigue, anorexia, nausea, cough, and joint pain. The remaining options are clinical manifestations that occur in the icteric phase. In the posticteric phase, jaundice decreases, the color of urine and stool returns to normal, and the client's appetite improves.

The nurse is caring for a client with possible cholelithiasis who is being prepared for intravenous cholangiography and is teaching the client about the procedure. Which statement indicates that the client understands the purpose of this test? 1. "My gallbladder will be irrigated." 2. "This procedure will drain my gallbladder." 3. "They will put medication in my gallbladder." 4. "They are going to look at my gallbladder and ducts."

4. "They are going to look at my gallbladder and ducts." Rationale: An intravenous cholangiogram is done for diagnostic purposes. It outlines both the gallbladder and the ducts, so gallstones that have moved into the ductal system can be detected. X-rays are used to visualize the biliary duct system after intravenous injection of radiopaque dye. This test is diagnostic and does not involve irrigation, instillation of medications, or drainage of the gallbladder. IGGY & Pagana Pagana

A nurse is participating in a health screening clinic and is preparing teaching materials about colorectal cancer. Which risk factor for colorectal cancer should the nurse include? 1. High-fiber, low-fat diet 2. Age older than 30 years 3. Distant relative with colorectal cancer 4. Personal history of ulcerative colitis or gastrointestinal polyps

4. Personal history of ulcerative colitis or gastrointestinal polyps Rationale: Common risk factors for colorectal cancer include age older than 40 years; first-degree relative with colorectal cancer; high-fat, low-fiber diet; and history of bowel problems, such as ulcerative colitis or familial polyposis.

A client suspected of having an abdominal tumor is scheduled for a computed tomography (CT) scan with dye injection. How should the nurse describe this test to the client? 1. The test may be painful. 2. The test will take approximately 2 hours. 3. Fluids will be restricted following the test. 4. The dye injected may cause a warm flushing sensation.

4. The dye injected may cause a warm flushing sensation. Rationale: CT scanning causes no pain and can take 15 to 60 minutes to perform. The dye may cause a warm flushing sensation when injected. Fluids are encouraged following the procedure. If an iodine dye is used, the client should be asked about allergies to seafood or iodine. Pagana Pagana

Jerod is experiencing an acute episode of ulcerative colitis. Which is priority for this patient? A. Replace lost fluid and sodium B. Monitor for increased serum glucose level from steroid therapy C. Restrict the dietary intake of foods high in potassium D. Note any change in the color and consistency of stools

A. Replace lost fluid and sodium Explanation: Diarrhea d/t an acute episode of ulcerative colitis leads to fluid & electrolyte losses so fluid replacement takes priority.

George has a T tube in place after gallbladder surgery. Before discharge, what information or instructions should be given regarding the T tube drainage? A. "If there is any drainage, notify the surgeon immediately." B. "The drainage will decrease daily until the bile duct heals." C. "First, the drainage is dark green; then it becomes dark yellow." D. "If the drainage stops, milk the tube toward the puncture wound."

B. "The drainage will decrease daily until the bile duct heals." Explanation: As healing occurs from the bile duct, bile drains from the tube; the amount of bile should decrease. Teach the patient to expect dark green drainage and to notify the doctor if drainage stops.

Your patient, Christopher, has a diagnosis of ulcerative colitis and has severe abdominal pain aggravated by movement, rebound tenderness, fever, nausea, and decreased urine output. This may indicate which complication? A. Fistula B. Bowel perforation C. Bowel obstruction D. Abscess

B. Bowel perforation Explanation: An inflammatory condition that affects the surface of the colon, ulcerative colitis causes friability and erosions with bleeding. Patients with ulcerative colitis are at increased risk for bowel perforation, toxic megacolon, hemorrhage, cancer, and other anorectal and systemic complications.

You're caring for Lewis, a 67 y.o. patient with liver cirrhosis who developed ascites and requires paracentesis. Relief of which symptom indicated that the paracentesis was effective? A. Pruritus B. Dyspnea C. Jaundice D. Peripheral Neuropathy

B. Dyspnea Explanation: Ascites puts pressure on the diaphragm. Paracentesis is done to remove fluid and reducing pressure on the diaphragm. The goal is to improve the patient's breathing. The others are signs of cirrhosis that aren't relieved by paracentesis

Ralph has a history of alcohol abuse and has acute pancreatitis. Which lab value is most likely to be elevated? A. Calcium B. Glucose C. Magnesium D. Potassium

B. Glucose Explanation: Glucose level increases and diabetes mellitus may result d/t the pancreatic damage to the islets of langerhans

Claire, a 33 y.o. is on your floor with a possible bowel obstruction. Which intervention is priority for her? A. Obtain daily weights B. Measure abdominal girth C. Keep strict intake and output D. Encourage her to increase fluids

B. Measure abdominal girth Explanation: Measuring abdominal girth provides quantitative information about increases or decreases in the amount of distention

The client with a duodenal ulcer may exhibit which of the following findings on assessment? A. Hematemesis B. Malnourishment C. Melena D. Pain with eating

C. Melena Explanation: The client with a duodenal ulcer may have bleeding at the ulcer site, which shows up as melena (black tarry poop). The other findings are consistent with a gastric ulcer.

Your patient Maria takes NSAIDS for her degenerative joint disease, has developed peptic ulcer disease. Which drug is useful in preventing NSAID-induced peptic ulcer disease? A. Calcium carbonate (Tums) B. Famotidine (Pepcid) C. Misoprostol (Cytotec) D. Sucralfate (Carafate)

C. Misoprostol (Cytotec) Explanation: Misoprostol restores prostaglandins that protect the stomach from NSAIDS, which diminish the prostaglandins

Donald is a 61 y.o. man with diverticulitis. Diverticulitis is characterized by: A. Periodic rectal hemorrhage B. Hypertension and tachycardia C. Vomiting and elevated temperature D. Crampy and lower left quadrant pain and low-grade fever

D. Crampy and lower left quadrant pain and low-grade fever Explanation: One sign of acute diverticulitis is crampy lower left quadrant pain. A low-grade fever is another common sign.

Which of the following tests can be used to diagnose ulcers? A. Abdominal x-ray B. Barium swallow C. Computed tomography (CT) scan D. Esophagogastroduodenoscopy (EGD)

D. Esophagogastroduodenoscopy (EGD) Explanation: The EGD can visualize the entire upper GI tract as well as allow for tissue specimens and electrocautery if needed. The barium swallow could locate a gastric ulcer. A CT scan and an abdominal x-ray aren't useful in the diagnosis of an ulcer.

A client is experiencing blockage of the common bile duct. The nurse anticipates that the client's diet will be altered because the client will experience difficulty digesting which nutrient? 1. Fats 2. Proteins 3. Carbohydrates 4. Water-soluble vitamins

1. Fats Rationale: Blockage of the common bile duct impedes the flow of bile from the gallbladder to the duodenum. Bile acids or bile salts are produced by the liver to emulsify or break down fats. Options 2, 3, and 4 are incorrect. IGGY

A client is admitted to the hospital with a diagnosis of acute diverticulitis. What should the nurse expect to be prescribed for this client? 1. NPO (nothing by mouth) status 2. Ambulation at least four times daily 3. Cholinergic medications to reduce pain 4. Coughing and deep breathing every 2 hours

1. NPO (nothing by mouth) status Rationale: During the acute phase of diverticulitis, the goal of treatment is to rest the bowel and allow the inflammation to subside. The client remains NPO and is placed on bed rest. Pain occurs from bowel spasms, and increased intra-abdominal pressure (coughing and deep breathing) may precipitate an attack. Ambulation and cholinergics will increase peristalsis. The remaining options are not interventions for the client with acute diverticulitis.

A client has been taking omeprazole (Prilosec) for 4 weeks. The ambulatory care nurse evaluates that the client is receiving the optimal intended effect of the medication if the client reports the absence of which symptom? 1. Diarrhea 2. Heartburn 3. Flatulence 4. Constipation

2. Heartburn Rationale: Omeprazole is a proton pump inhibitor classified as an antiulcer agent. The intended effect of the medication is relief of pain from gastric irritation, often called heartburn by clients.

The nurse is assisting a client with Crohn's disease to ambulate to the bathroom. After the client has a bowel movement, the nurse should assess the stool for which characteristic that is expected with this disease? 1. Blood in the stool 2. Chalky gray stool 3. Loose, watery stool 4. Dry, hard, constipated stool

3. Loose, watery stool Rationale: Crohn's disease is characterized by nonbloody diarrhea of usually not more than four or five stools daily. Over time, the episodes of diarrhea increase in frequency, duration, and severity. IGGY

Which of the following definitions best describes gastritis? A. Erosion of the gastric mucosa B. Inflammation of a diverticulum C. Inflammation of the gastric mucosa D. Reflux of stomach acid into the esophagus

C. Inflammation of the gastric mucosa Explanation: Gastritis is an inflammation of the gastric mucosa that may be acute (often resulting from exposure to local irritants) or chronic (associated with autoimmune infections or atrophic disorders of the stomach). Erosion of the mucosa results in ulceration. Inflammation of a diverticulum is called diverticulitis; reflux of stomach acid is known as gastroesophageal disease.

Britney, a 20 y.o. student is admitted with acute pancreatitis. Which laboratory findings do you expect to be abnormal for this patient? A. Serum creatinine and BUN B. Alanine aminotransferase (ALT) and aspartate aminotransferase (AST) C. Serum amylase and lipase D. Cardiac enzymes

C. Serum amylase and lipase Explanation: Pancreatitis involves activation of pancreatic enzymes, such as amylase and lipase. These levels are elevated in a patient with acute pancreatitis.

A 29 y.o. patient has an acute episode of ulcerative colitis. What diagnostic test confirms this diagnosis? A. Barium Swallow. B. Stool examination C. Gastric analysis D. Colonoscopy

D. Colonoscopy Explanation: Colonoscopy allows direct observation of the colon mucosa for changes, and if needed, biopsy.

The nurse is assigned to a client receiving total parenteral nutrition (TPN) who had a blood glucose measurement done at 06:00. The nurse documents on the client's clinical worksheet for the day that the blood glucose level should be checked next at which time? 1. 08:00 2. 12:00 3. 16:00 4. 18:00

2. 12:00 Rationale: The client's blood glucose level should be monitored every 4 to 6 hours during TPN therapy. Depending on agency policy, this may be done every 8 hours instead. Monitoring the blood glucose level every 2 hours is unnecessary. Monitoring every 10 or 12 hours is insufficient.

A client with a history of gastrointestinal upset has been diagnosed with acute diverticulitis. The nurse should give the client suggestions for foods to aid in symptom management that are in which diet types? 1. A low-fat diet 2. A low-fiber diet 3. A high-protein diet 4. A high-carbohydrate diet

2. A low-fiber diet Rationale: A low-fiber diet places less strain on the intestines because this type of diet is easier to digest. Clients should avoid high-fiber foods when experiencing acute diverticulitis. As the attack resolves, fiber can be gradually added to the diet. IGGY

A nurse is reviewing the medication record of a client with acute gastritis. Which medication, if noted on the client's record, should the nurse question? 1. Digoxin (Lanoxin) 2. Furosemide (Lasix) 3. Indomethacin (Indocin) 4. Propranolol hydrochloride (Inderal LA)

3. Indomethacin (Indocin) Rationale: Indomethacin (Indocin) is a nonsteroidal anti-inflammatory drug and can cause ulceration of the esophagus, stomach, or small intestine. Indomethacin is contraindicated in a client with gastrointestinal disorders. Digoxin is a cardiac medication. Furosemide (Lasix) is a loop diuretic. Propranolol (Inderal) is a β-adrenergic blocker. Digoxin, furosemide, and propranolol are not contraindicated in clients with gastric disorders.

Which of the following tests can be performed to diagnose a hiatal hernia? A. Colonoscopy B. Lower GI series C. Barium swallow D. Abdominal x-rays

C. Barium swallow Explanation: A barium swallow with fluoroscopy shows the position of the stomach in relation to the diaphragm. A colonoscopy and a lower GI series show disorders of the intestine.

A client presents to the emergency department with upper gastrointestinal (GI) bleeding and is in moderate distress. In planning care, which nursing action should be the first priority for this client? 1. Assessment of vital signs 2. Complete abdominal examination 3. Thorough investigation of precipitating events 4. Insertion of a nasogastric tube and Hematest of emesis

1. Assessment of vital signs Rationale: The priority nursing action is to assess the vital signs. This would indicate the amount of blood loss that has occurred and also provides a baseline by which to monitor the progress of treatment. The client may not be able to provide subjective data until the immediate physical needs are met. Although an abdominal examination and an assessment of the precipitating events may be necessary, these actions are not the priority.

A nurse is reviewing the health care provider's prescriptions written for a client admitted to the hospital with acute pancreatitis. Which prescription should the nurse confirm? 1. Full liquid diet 2. Morphine sulfate for pain 3. Nasogastric tube insertion 4. An anticholinergic medication

1. Full liquid diet Rationale: The client with acute pancreatitis is placed on an NPO (nothing by mouth) status to decrease the activity of the pancreas, which occurs with oral intake. Pain management for acute pancreatitis typically begins with the administration of opioids by patient-controlled analgesia. Medications such as morphine or hydromorphone (Dilaudid) are typically used. Nasogastric tube insertion is done to provide suction of secretions and administer medications as necessary. IGGY

The nurse is caring for a client who is receiving total parenteral nutrition (TPN) via a central line. Which nursing intervention would specifically provide assessment data related to the most common complication associated with TPN? 1. Weighing the client daily 2. Monitoring the temperature 3. Monitoring intake and output (I&O) 4. Monitoring the blood urea nitrogen (BUN) level

2. Monitoring the temperature Rationale: The most common complication associated with TPN is infection. Monitoring the temperature provides assessment data that would indicate infection in the client.

A client with peptic ulcer disease who has been given a prescription for propantheline (Pro-Banthine) asks the nurse how to take the medication. The nurse explains that this medication should be taken in which way? 1. With meals 2. With antacids 3. Just after meals 4. 30 minutes before meals

4. 30 minutes before meals Rationale: Propantheline decreases gastrointestinal secretions because it is an antimuscarinic anticholinergic medication. It should be administered 30 minutes before meals. Kee Hayes

The nurse is providing dietary instructions to a client hospitalized for pancreatitis. Which food should the nurse instruct the client to avoid? 1. Chili 2. Bagel 3. Lentil soup 4. Watermelon

1. Chili Rationale: The client with pancreatitis needs to avoid alcohol, coffee and tea, spicy foods, and heavy meals, which stimulate pancreatic secretions, producing attacks of pancreatitis. The client is instructed in the benefit of eating small, frequent meals that are high in protein, low in fat, and moderate to high in carbohydrates. IGGY

Which finding suggests to the nurse that a client with bleeding esophageal varices is experiencing an adverse effect of vasopressin therapy? 1. Complaints of chest pain 2. Temperature of 102° F 3. Bounding peripheral pulses 4. Blood urea nitrogen (BUN) of 20 mg/dL

1. Complaints of chest pain Rationale: Vasopressin therapy causes vasoconstriction, and adverse effects include myocardial ischemia, which may be evident by the client's complaints of chest pain. Options 2, 3, and 4 are not adverse effects. Vasopressin therapy can cause hypothermia. Because vasopressin has potent vasoconstrictive effects on the peripheral arterioles, weak versus bounding pulses may be found. The BUN noted in option 4 is normal. IGGY

The nurse is monitoring a client who required a Sengstaken-Blakemore tube because other measures for treating bleeding esophageal varices were unsuccessful. The client complains of severe pain of abrupt onset. Which nursing action is most appropriate? 1. Cut the tube. 2. Reposition the client. 3. Assess the lumens of the tubes. 4. Administer the prescribed analgesics.

1. Cut the tube. Rationale: Spontaneous rupture of the gastric balloon, upward migration of the tube, and occlusion of the airway are possible complications associated with a Sengstaken-Blakemore tube. Esophageal rupture also may occur and is characterized by the abrupt onset of severe pain. In the event of either of these life-threatening emergencies, the tube is cut and removed.

A nurse is caring for a client diagnosed with suspected acute pancreatitis. When reviewing the client's laboratory results, the nurse interprets that which finding will support the diagnosis? 1. Elevated serum lipase level 2. Elevated serum bilirubin level 3. Decreased serum trypsin level 4. Decreased serum amylase level

1. Elevated serum lipase level Rationale: The serum lipase level is elevated in the presence of pancreatic cell injury. Serum trypsin and amylase levels are also elevated in pancreatic injury. Although bilirubin can be elevated in the client with pancreatitis, it is secondary to the hepatobiliary obstructive process. IGGY

A client is admitted to the hospital with acute viral hepatitis. Which sign or symptom should the nurse expect to note based on this diagnosis? 1. Fatigue 2. Pale urine 3. Weight gain 4. Spider angiomas

1. Fatigue Rationale: Common manifestations of acute viral hepatitis include weight loss, dark urine, and fatigue. The client is anorexic, possibly from a toxin produced by the diseased liver, and finds food distasteful. The urine darkens because of excess bilirubin being excreted by the kidneys. Fatigue occurs during all phases of hepatitis. Spider angiomas, small, dilated blood vessels, are commonly seen in cirrhosis of the liver.

A client with a history of gastric ulcer complains of a sudden, sharp, severe pain in the midepigastric area, which then spreads over the entire abdomen. The client's abdomen is rigid and boardlike on palpation, and the client obtains most comfort from lying in the knee-chest position. The nurse calls the health care provider immediately, suspecting that the client is experiencing which complication of peptic ulcer disease? 1. Perforation 2. Obstruction 3. Hemorrhage 4. Intractability

1. Perforation Rationale: The signs and symptoms described in the question are consistent with perforation of the ulcer, which may progress to peritonitis if the perforation is large enough. A client with intestinal obstruction would most likely complain of abdominal pain and distention and nausea and vomiting. A client with hemorrhage would vomit blood or coffee-ground material or would expel black, tarry, or bloody stools. Intractability is a term that refers to continued manifestations of a disease process despite ongoing medical treatment. IGGY

A client presents to the urgent care center with complaints of abdominal pain and vomits bright red blood. Which is the priority nursing action? 1. Take the client's vital signs. 2. Perform a complete abdominal assessment. 3. Obtain a thorough history of the recent health status. 4. Prepare to insert a nasogastric tube and test pH and occult blood.

1. Take the client's vital signs. Rationale: The nurse should take the client's vital signs first to determine whether he or she is hypovolemic or in shock from blood loss. This also provides a baseline blood pressure and pulse by which to gauge the effectiveness of treatment. Signs and symptoms of shock include low blood pressure, rapid weak pulse, increased thirst, cold clammy skin, and restlessness. Therefore the remaining options are incorrect. IGGY

A nurse is teaching the postgastrectomy client about measures to prevent dumping syndrome. Which statement by the client indicates a need for further teaching? 1. "I need to lie down after eating." 2. "I need to drink liquids with meals." 3. "I need to avoid concentrated sweets." 4. "I need to eat small meals six times daily."

2. "I need to drink liquids with meals." Rationale: The client with dumping syndrome should avoid drinking liquids with meals. The client should be placed on a high-protein, moderate-fat, and high-calorie diet and should lie down after eating. The client should avoid concentrated sweets and frequent small meals are encouraged.

A home care nurse visits a client who was recently diagnosed with cirrhosis. The nurse provides home care management instructions to the client. Which client statement indicates a need for further instruction? 1. "I will obtain adequate rest." 2. "I will take Tylenol if I get a headache." 3. "I should monitor my weight on a regular basis." 4. "I need to include sufficient amounts of carbohydrates in my diet."

2. "I will take Tylenol if I get a headache." Rationale: Acetaminophen is avoided because it can cause fatal liver damage in the client with cirrhosis. Adequate rest and nutrition are important. The client's weight should be monitored on a regular basis. The diet should supply sufficient carbohydrates with a total daily calorie intake of 2000 to 3000. IGGY

A client with acute pancreatitis is experiencing severe pain from the disorder. The nurse determines that the client understands suggestions for positioning to reduce pain if he or she avoids which action? 1. Sitting up 2. Lying flat 3. Leaning forward 4. Drawing the legs up to the chest

2. Lying flat Rationale: The pain of pancreatitis is aggravated by lying supine or walking. This is because the pancreas is located retroperitoneally, and the edema and inflammation will intensify the irritation of the posterior peritoneal wall with these positions or movements. Positions such as sitting up, leaning forward, and flexing the legs (especially the left leg) will alleviate some of the pain associated with pancreatitis. The fetal position (with the legs drawn up to the chest) may decrease the abdominal pain of pancreatitis.

A client is readmitted to the hospital with dehydration after surgery for creation of an ileostomy. The nurse assesses that the client has lost 3 lb of weight, has poor skin turgor, and has concentrated urine. The nurse interprets the client's clinical picture as correlating most closely with recent intake of which medication, which is contraindicated for the ileostomy client? 1. Folate (folic acid) 2. Sennosides (Ex-Lax) 3. Ferrous sulfate (Feosol) 4. Cyanocobalamin (vitamin B12)

2. Sennosides (Ex-Lax) Rationale: The client with an ileostomy is prone to dehydration because of the location of the ostomy in the gastrointestinal tract and should not take laxatives. Laxatives will compound the potential risk for the client. These clients are at risk for deficiencies of folate, iron, and cyanocobalamin and should receive them as supplements if necessary. Kee Hayes

A client is taking lansoprazole (Prevacid) for the chronic management of peptic ulcer disease. The nurse expects that the health care provider will advise the client to take which product if needed for a headache? 1. Naproxen (Aleve) 2. Ibuprofen (Motrin IB) 3. Acetaminophen (Tylenol) 4. Acetylsalicylic acid (aspirin)

3. Acetaminophen (Tylenol) Rationale: The client with peptic ulcer disease should avoid taking medications that are irritating to the stomach lining. Irritants would include aspirin and nonsteroidal anti-inflammatory drugs (NSAIDs). The client should be advised to take acetaminophen for a headache. IGGY

A client with portal-systemic encephalopathy is receiving oral lactulose (Chronulac) daily. The nurse should check which item to determine the effectiveness of this medication? 1. Lung sounds 2. Blood pressure 3. Blood ammonia level 4. Serum potassium level

3. Blood ammonia level Rationale: Lactulose is a hyperosmotic laxative and ammonia detoxicant. It is used to prevent or treat portal-systemic encephalopathy, including hepatic precoma and coma. It also is used to treat constipation. The medication retains ammonia in the colon (decreases the blood ammonia concentration), producing an osmotic effect. It promotes increased peristalsis and bowel evacuation, expelling ammonia from the colon. This medication has no effect on lung sounds, the blood pressure, or the serum potassium level.

The medication history of a client with peptic ulcer disease reveals intermittent use of several medications. The nurse would teach the client to avoid which of these medications because of its irritating effects on the lining of the gastrointestinal tract? 1. Nizatidine (Axid) 2. Sucralfate (Carafate) 3. Ibuprofen (Motrin IB) 4. Omeprazole (Prilosec)

3. Ibuprofen (Motrin IB) Rationale: Ibuprofen is a nonsteroidal anti-inflammatory drug that typically is irritating to the lining of the gastrointestinal tract and should be avoided by clients with a history of peptic ulcer disease. The other medications listed are frequently used to treat peptic ulcer disease. Nizatidine is an H2-receptor antagonist that reduces the secretion of gastric acid. Sucralfate coats the surface of an ulcer to promote healing. Omeprazole is a proton-pump inhibitor that blocks transport of hydrogen ions into the lumen of the gastrointestinal tract.

A nurse is reviewing laboratory test results for the client with liver disease and notes that the client's albumin level is low. Which nursing action is focused on the consequence of low albumin levels? 1. Evaluating for asterixis 2. Inspecting for petechiae 3. Palpating for peripheral edema 4. Evaluating for decreased level of consciousness

3. Palpating for peripheral edema Rationale: Albumin is responsible for maintaining the osmolality of the blood. When there is a low albumin level, there is decreased osmotic pressure, which in turn can lead to peripheral edema. Options 1, 2, and 4 are incorrect and are not associated with a low albumin level. IGGY

A nurse is caring for a hospitalized client with a diagnosis of ulcerative colitis. Which finding, if noted on assessment of the client, should the nurse report to the health care provider (HCP)? 1. Hypotension 2. Bloody diarrhea 3. Rebound tenderness 4. A hemoglobin level of 12 mg/dL

3. Rebound tenderness Rationale: Rebound tenderness may indicate peritonitis. Bloody diarrhea is expected to occur in ulcerative colitis. Because of the blood loss, the client may be hypotensive and the hemoglobin level may be lower than normal. Signs of peritonitis must be reported to the HCP. IGGY

A nurse is reviewing the preoperative prescriptions for a client with a colon tumor who is scheduled for abdominal perineal resection and notes that the health care provider has prescribed neomycin (Mycifradin) for the client. The nurse determines that this medication has been prescribed primarily for which purpose? 1. To prevent an immune dysfunction 2. Because the client has an infection 3. To decrease the bacteria in the bowel 4. Because the client is allergic to penicillin

3. To decrease the bacteria in the bowel Rationale: To reduce the risk of contamination at the time of surgery, the surgeon may prescribe that the bowel is emptied and cleansed. Laxatives and enemas are given to empty the bowel. Intestinal anti-infectives such as neomycin or kanamycin are administered to decrease the bacteria in the bowel. The medication does not prevent immune dysfunction. There is no data in the question that indicates that the client has an infection or is allergic to penicillin. IGGY

The client with a small bowel obstruction asks the nurse to explain the purpose of the nasogastric tube attached to continuous gastric suction. The nurse determines that teaching has been effective if the client makes which statement? 1. "It will help to provide me nourishment." 2. "It will help to relieve the congestion from excess mucus." 3. "It is used to remove gastric contents for laboratory analysis." 4. "It will help to remove gas and fluids from my stomach and intestine."

4. "It will help to remove gas and fluids from my stomach and intestine." Rationale: Treatment of intestinal obstruction is directed toward decompression of the intestine by removal of gas and fluid. Nasogastric tubes may be used to decompress the stomach and bowel. Continuous gastric suction does not provide nourishment. The purpose of tracheal suctioning (not gastric suctioning) is to remove excess mucus that has led to congestion. Although gastric contents may be sent for laboratory analysis, it is not the main purpose for continuous gastric suction.

The nurse is performing an admission assessment on a client who has been admitted to the hospital with a diagnosis of suspected gastric ulcer. The nurse is asking the client questions about pain. Which statement, if made by the client, would support the diagnosis of gastric ulcer? 1. "The pain doesn't usually come right after I eat." 2. "The pain gets so bad that it wakes me up at night." 3. "The pain that I get is located on the right side of my chest." 4. "My pain comes shortly after I eat, maybe a half-hour or so later."

4. "My pain comes shortly after I eat, maybe a half-hour or so later." Rationale: Gastric ulcer pain often occurs in the upper epigastrium, with localization to the left of the midline, and may be exacerbated by intake of food. The pain occurs 30 to 60 minutes after a meal and rarely occurs at night. Duodenal ulcer pain is usually located to the right of the epigastrium. The pain associated with a duodenal ulcer occurs 90 minutes to 3 hours after eating and often awakens the client at night. IGGY

A client with gastritis asks the nurse at a screening clinic about analgesics that will not cause epigastric distress. The nurse should tell the client that which medication is unlikely to cause epigastric distress? 1. Ecotrin 2. Bufferin 3. Ascriptin 4. Acetaminophen (Tylenol)

4. Acetaminophen (Tylenol) Rationale: Aspirin is irritating to the gastrointestinal (GI) tract of the client with a history of gastritis. Analgesics that do not contain aspirin, such as acetaminophen, are unlikely to cause gastric distress. The other medications listed have aspirin in them. Another category of medications that is irritating to the GI tract is the nonsteroidal anti-inflammatory drugs.

A client is scheduled for an upper gastrointestinal (GI) endoscopy. Which assessment is essential to include in the plan of care following the procedure? 1. Assessing pulses 2. Monitoring urine output 3. Monitoring for rectal bleeding 4. Assessing for the presence of the gag reflex

4. Assessing for the presence of the gag reflex Rationale: Following the procedure, the client remains NPO until the gag reflex returns, which is usually in 1 to 2 hours. The remaining options are not specific assessments related to this procedure. Pagana Pagana

A nurse is preparing to care for a client following a gastroscopy procedure. Which priority component should the nurse include in the nursing care plan? 1. Monitor the client's vital signs every hour for 4 hours. 2. Place the client in a supine position to provide comfort. 3. Provide saline gargles immediately on return to the unit to aid in comfort. 4. Check the gag reflex by using a tongue depressor to stroke the back of the client's throat.

4. Check the gag reflex by using a tongue depressor to stroke the back of the client's throat. Rationale: Before the gastroscopy procedure medication is given to prevent a gag reflex. On return from the procedure, the nurse must test the client's gag reflex to ensure that it is present to prevent aspiration of contents. Vital signs should be taken every 30 minutes for 2 hours to detect abnormalities. The client must be placed in a side-lying or semi-Fowler's position to avoid aspiration. Saline gargles must only be administered when the gag reflex has been confirmed. Pagana Pagana

The nurse should instruct a client with an ileostomy to include which action as part of essential care of the stoma? 1. Massage the area below the stoma. 2. Take in high-fiber foods such as nuts. 3. Limit fluid intake to prevent diarrhea. 4. Cleanse the peristomal skin meticulously.

4. Cleanse the peristomal skin meticulously. Rationale: The peristomal skin must receive meticulous cleansing because ileostomy drainage has more enzymes and is more caustic to the skin than colostomy drainage. The area below the ileostomy may be massaged as needed if the ileostomy becomes blocked by high-fiber foods. Foods such as nuts and those with seeds will pass through the ileostomy. The client should be taught that these foods will remain undigested. Fluid intake should be at least six to eight glasses of water per day to prevent dehydration.

A client with cirrhosis complicated by ascites is admitted to the hospital. The client reports a 10-lb weight gain over the last 1½ weeks. The client has edema of the feet and ankles, and his abdomen is distended, taut, and shiny with striae. Which client problem is most appropriate at this time? 1. Difficulty with breathing 2. Risk for skin breakdown 3. Difficulty with sleeping 4. Excessive body fluid volume

4. Excessive body fluid volume Rationale: The client with weight gain who also has cirrhosis complicated by ascites most often is retaining fluid. This is especially true when the client has not demonstrated an appreciable increase in food intake or when the weight gain is massive in relation to the time frame given. Therefore, excessive body fluid volume is the most appropriate problem. No data are given to support difficulty with breathing, although in some clients upward pressure on the diaphragm from ascites does impair respiration. Risk for skin breakdown assumes a lower priority because it is a risk rather than an actual problem. There is no data in the question that indicates that the client is having difficulty with sleep.

The nurse provides medication instructions to a client with peptic ulcer disease. Which statement, if made by the client, indicates the best understanding of the medication therapy? A. "The cimetidine (Tagamet) will cause me to produce less stomach acid." B. "Sucralfate (Carafate) will change the fluid in my stomach." C. "Antacids will coat my stomach." D. "Omeprazole (Prilosec) will coat the ulcer and help it heal."

A. "The cimetidine (Tagamet) will cause me to produce less stomach acid." Explanation: Cimetidine (Tagamet), a histamine H2 receptor antagonist, will decrease the secretion of gastric acid. Sucralfate (Carafate) promotes healing by coating the ulcer. Antacids neutralize acid in the stomach. Omeprazole (Prilosec) inhibits gastric acid secretion.

A female client who has just been diagnosed with hepatitis A asks, "How could I have gotten this disease?" What is the nurse's best response? A. "You may have eaten contaminated restaurant food." B. "You could have gotten it by using I.V. drugs." C. "You must have received an infected blood transfusion." D. "You probably got it by engaging in unprotected sex."

A. "You may have eaten contaminated restaurant food." Explanation: Hepatitis A virus typically is transmitted by the oral-fecal route — commonly by consuming food contaminated by infected food handlers. The virus isn't transmitted by the I.V. route, blood transfusions, or unprotected sex. Hepatitis B can be transmitted by I.V. drug use or blood transfusion. Hepatitis C can be transmitted by unprotected sex.

You are developing a care plan on Sally, a 67 y.o. patient with hepatic encephalopathy. Which of the following do you include? A. Administering a lactulose enema as ordered B. Encouraging a protein-rich diet C. Administering sedatives, as necessary D. Encouraging ambulation at least four times a day

A. Administering a lactulose enema as ordered Explanation: You may administer the laxative lactulose to reduce ammonia levels in the colon.

Fistulas are most common with which of the following bowel disorders? A. Crohn's disease B. Diverticulitis C. Diverticulosis D. Ulcerative colitis

A. Crohn's disease Explanation: The lesions of Crohn's disease are transmural; that is, they involve all thickness of the bowel. These lesions may perforate the bowel wall, forming fistulas with adjacent structures. Fistulas don't develop in diverticulitis or diverticulosis. The ulcers that occur in the submucosal and mucosal layers of the intestine in ulcerative colitis usually don't progress to fistula formation as in Crohn's disease.

Glenda has cholelithiasis (gallstones). You expect her to complain of: A. Pain in the right upper quadrant, radiating to the shoulder B. Pain in the right lower quadrant, with rebound tenderness C. Pain in the left upper quadrant, with shortness of breath D. Pain in the left lower quadrant, with mild cramping

A. Pain in the right upper quadrant, radiating to the shoulder Explanation: The gallbladder is located in the RUQ and a frequent sign of gallstones is pain radiating to the shoulder

A client has been taking Ibuprofen for some quite time and was given Misoprostol (Cytotec). Which of the following is exhibiting the therapeutic effect of Cytotec? A. Relief of gastric ulcer. B. Relief of diarrhea. C. Relief of vomiting. D. Relief of constipation.

A. Relief of gastric ulcer. Explanation: Misoprostol (Cytotec) is a synthetic (man-made) prostaglandin that is used to reduce the risk of stomach ulcers in patients treated with nonsteroidal antiinflammatory drugs (NSAIDs, for example, aspirin, ibuprofen, etc.). Option B: Although a side effect, but it is not the intended therapeutic effect. Options C and D are not related to the medication.

A patient has an acute upper GI hemorrhage. Your interventions include: A. Treating hypovolemia B. Treating hypervolemia C. Controlling the bleeding source D. Treating shock and diagnosing the bleeding source

A. Treating hypovolemia Explanation: A patient with an acute upper GI hemorrhage must be treated for hypovolemia and hemorrhagic shock. You as a nurse can't diagnose the problem. Controlling the bleeding may require surgery or intensive medical treatmen

A male client with cholelithiasis has a gallstone lodged in the common bile duct. When assessing this client, the nurse expects to note: A. yellow sclera B. light amber urine C. circumoral pallor D. black, tarry stools

A. yellow sclera Explanation: Yellow sclerae may be the first sign of jaundice, which occurs when the common bile duct is obstructed. Urine normally is light amber. Circumoral pallor and black, tarry stools don't occur in common bile duct obstruction; they are signs of hypoxia and GI bleeding, respectively.

A client with a duodenal ulcer is diagnosed with H. pylori infection. The physician prescribed Amoxicillin (Wymox), Pantoprazole (Prevacid), and Clarithromycin (Biaxin). Which statement made by the nurse correctly explains the purpose of these medications? A. "These medicines will minimize acid production and will coat the ulcer". B. "These medicines will stop the acid production and will kill the bacteria" C. "The ulcer will heal because the medications will kill the bacteria". D. "These medicines will control the ulcer and motion sickness".

B. "These medicines will stop the acid production and will kill the bacteria" Explanation: The triple therapy treatment of H-pylori infection is the includes 2 antibiotics (Clarithromycin and Amoxicillin) and one proton pump inhibitor such as omeprazole, lansoprazole, pantoprazole, or esomeprazole.

Which of the following symptoms is a client with colon cancer most likely to exhibit? A. A change in appetite B. A change in bowel habit C. An increase in body weight D. An increase in body temperature

B. A change in bowel habit Explanation: The most common complaint of the client with colon cancer is a change in bowel habits. The client may have anorexia, secondary abdominal distention, or weight loss. Fever isn't associated with colon cancer.

Which of the following definitions best describes diverticulosis? A. An inflamed outpouching of the intestine B. A noninflamed outpouching of the intestine C. The partial impairment of the forward flow of intestinal contents D. An abnormal protrusion of an organ through the structure that usually holds it.

B. A noninflamed outpouching of the intestine Explanation: Diverticulosis involves a noninflamed outpouching of the intestine. Diverticulitis involves an inflamed outpouching. The partial impairment of forward flow of the intestine is an obstruction; abnormal protrusion of an organ is a hernia.

Which of the following tests is most commonly used to diagnose cholecystitis? A. Abdominal CT scan B. Abdominal ultrasound C. Barium swallow D. Endoscopy

B. Abdominal ultrasound Explanation: An abdominal ultrasound can show if the gallbladder is enlarged, if gallstones are present, if the gallbladder wall is thickened, or if distention of the gallbladder lumen is present. An abdominal CT scan can be used to diagnose cholecystitis, but it usually isn't necessary. A barium swallow looks at the stomach and the duodenum. Endoscopy looks at the esophagus, stomach, and duodenum.

Which of the following tests should be administered to a client suspected of having diverticulosis? A. Abdominal ultrasound B. Barium enema C. Barium swallow D. Gastroscopy

B. Barium enema Explanation: A barium enema will cause diverticula to fill with barium and be easily seen on x-ray. An abdominal US can tell more about structures, such as the gallbladder, liver, and spleen, than the intestine. A barium swallow and gastroscopy view upper GI structures.

Regina is a 46 y.o. woman with ulcerative colitis. You expect her stools to look like: A. Watery and frothy B. Bloody and mucous C. Firm and well-formed D. Alternating constipation and diarrhea

B. Bloody and mucous Explanation: Stools from ulcerative colitis are often bloody and contain mucus.

Findings during an endoscopic exam include a cobblestone appearance of the colon in your patient. The findings are characteristic of which disorder? A. Ulcer B. Crohn's disease C. Chronic gastritis D. Ulcerative colitis

B. Crohn's disease Explanation: Crohn's disease penetrates the mucosa of the colon through all layers and destroys the colon in patches, which creates a cobblestone appearance.

A client is receiving Sulfasalazine (Azulfidine) for the treatment of ulcerative colitis. Which of the following assessment finding will concern the nurse most? A. Drowsiness. B. Decreased urine output. C. Urine discoloration. D. Vomiting.

B. Decreased urine output. Explanation: Sulfasalazine is used to treat bowel inflammation, diarrhea (stool frequency), rectal bleeding, and abdominal pain in patients with ulcerative colitis. It is nephrotoxic, so a decrease in urine output is the most serious concern.

Which of the following best describes the method of action of medications, such as ranitidine (Zantac), which are used in the treatment of peptic ulcer disease? A. Neutralize acid B. Reduce acid secretions C. Stimulate gastrin release D. Protect the mucosal barrier

B. Reduce acid secretions Explanation: Ranitidine is a histamine-2 receptor antagonist that reduces acid secretion by inhibiting gastrin secretion.

A client has been diagnosed with adenocarcinoma of the stomach and is scheduled to undergo a subtotal gastrectomy (Billroth II procedure). During pre-operative teaching, the nurse is reinforcing information about the procedure. Which of the following explanations is most accurate? A. The procedure will result in enlargement of the pyloric sphincter B. The procedure will result in anastomosis of the gastric stump to the jejunum C. The procedure will result in removal of the duodenum D. The procedure will result in repositioning of the vagus nerve

B. The procedure will result in anastomosis of the gastric stump to the jejunum Explanation: A Billroth II procedure bypasses the duodenum and connects the gastric stump directly to the jejunum. The pyloric sphincter is removed, along with some of the stomach fundus.

When evaluating a male client for complications of acute pancreatitis, the nurse would observe for: A. increased intracranial pressure B. decreased urine output C. bradycardia D. hypertension

B. decreased urine output Explanation: Acute pancreatitis can cause decreased urine output, which results from the renal failure that sometimes accompanies this condition. Intracranial pressure neither increases nor decreases in a client with pancreatitis. Tachycardia, not bradycardia, usually is associated with pulmonary or hypovolemic complications of pancreatitis. Hypotension can be caused by a hypovolemic complication, but hypertension usually isn't related to acute pancreatitis.

Medical management of the client with diverticulitis should include which of the following treatments? A. Reduced fluid intake B. Increased fiber in diet C. Administration of antibiotics D. Exercises to increase intra-abdominal pressure

C. Administration of antibiotics Explanation: Antibiotics are used to reduce the inflammation. The client isn't typically isn't allowed anything orally until the acute episode subsides. Parenteral fluids are given until the client feels better; then it's recommended that the client drink eight 8-ounce glasses of water per day and gradually increase fiber in the diet to improve intestinal motility. During the acute phase, activities that increase intra-abdominal pressure should be avoided to decrease pain and the chance of intestinal obstruction.

Which of the following symptoms is common with a hiatal hernia? A. Left arm pain B. Lower back pain C. Esophageal reflux D. Abdominal cramping

C. Esophageal reflux Explanation: Esophageal reflux is a common symptom of hiatal hernia. This seems to be associated with chronic exposure of the lower esophageal sphincter to the lower pressure of the thorax, making it less effective.

The nurse is caring for a female client following a Billroth II procedure. Which postoperative order should the nurse question and verify? A. Leg exercises B. Early ambulation C. Irrigating the nasogastric tube D. Coughing and deep-breathing exercises

C. Irrigating the nasogastric tube Explanation: In a Billroth II procedure, the proximal remnant of the stomach is anastomosed to the proximal jejunum. Patency of the nasogastric tube is critical for preventing the retention of gastric secretions. The nurse should never irrigate or reposition the gastric tube after gastric surgery, unless specifically ordered by the physician. In this situation, the nurse should clarify the order. Options A, B, and D are appropriate postoperative interventions.

The most important pathophysiologic factor contributing to the formation of esophageal varices is: A. Decreased prothrombin formation B. Decreased albumin formation by the liver C. Portal hypertension D. Increased central venous pressure

C. Portal hypertension Explanation: As the liver cells become fatty and degenerate, they are no longer able to accommodate the large amount of blood necessary for homeostasis. The pressure in the liver increases and causes increased pressure in the venous system. As the portal pressure increases, fluid exudes into the abdominal cavity. This is called ascites.

A 53 y.o. patient has undergone a partial gastrectomy for adenocarcinoma of the stomach. An NG tube is in place and is connected to low continuous suction. During the immediate postoperative period, you expect the gastric secretions to be which color? A. Brown B. Clear C. Red D. Yellow

C. Red Explanation: Normally, drainage is bloody for the first 24 hours after a partial gastrectomy; then it changes to brown-tinged and then to yellow or clear

A male client with a peptic ulcer is scheduled for a vagotomy and the client asks the nurse about the purpose of this procedure. Which response by the nurse best describes the purpose of a vagotomy? A. Halts stress reactions B. Heals the gastric mucosa C. Reduces the stimulus to acid secretions D. Decreases food absorption in the stomach

C. Reduces the stimulus to acid secretions Explanation: A vagotomy, or cutting of the vagus nerve, is done to eliminate parasympathetic stimulation of gastric secretion.

The nurse is giving instructions to a client receiving Cholestyramine (Prevalite). Which statement made by the client indicates a need for further teachings? A. "I will include a high fiber rich food in my diet". B. "I will continue taking my multivitamins". C. "This medication will help lower my cholesterol". D. "I will sip the cholestyramine powder for a long time for faster absorption".

D. "I will sip the cholestyramine powder for a long time for faster absorption". Explanation: Cholestyramine is a bile acid sequestrant. It works by helping the body remove bile acids, which can lower cholesterol levels in the blood. A powder form of this medication can cause tooth problems such as discoloration, erosion of enamel, or decay. Other side effects of this medication are decreased vitamin absorption and constipation.

A client is taking an antacid for treatment of a peptic ulcer. Which of the following statements best indicates that the client understands how to correctly take the antacid? A. "I should take my antacid before I take my other medications." B. "I need to decrease my intake of fluids so that I don't dilute the effects of my antacid." C. "My antacid will be most effective if I take it whenever I experience stomach pains." D. "It is best for me to take my antacid 1 to 3 hours after meals."

D. "It is best for me to take my antacid 1 to 3 hours after meals." Explanation: Antacids are most effective if taken 1 to 3 hours after meals and at bedtime. When an antacid is taken on an empty stomach, the duration of the drug's action is greatly decreased. Taking antacids 1 to 3 hours after a meal lengthens the duration of action, thus increasing the therapeutic action of the drug. Antacids should be administered about 2 hours after other medications to decrease the chance of drug interactions. It is not necessary to decrease fluid intake when taking antacids.

If a client had irritable bowel syndrome, which of the following diagnostic tests would determine if the diagnosis is Crohn's disease or ulcerative colitis? A. Abdominal computed tomography (CT) scan B. Abdominal x-ray C. Barium swallow D. Colonoscopy with biopsy

D. Colonoscopy with biopsy Explanation: A colonoscopy with biopsy can be performed to determine the state of the colon's mucosal layers, presence of ulcerations, and level of cytologic development. An abdominal x-ray or CT scan wouldn't provide the cytologic information necessary to diagnose which disease it is. A barium swallow doesn't involve the intestine

Your patient has a retractable gastric peptic ulcer and has had a gastric vagotomy. Which factor increases as a result of vagotomy? A. Peristalsis B. Gastric acidity C. Gastric motility D. Gastric pH

D. Gastric pH Explanation: If the vagus nerve is cut as it enters the stomach, gastric acid secretion is decreased, but intestinal motility is also decreased and gastric emptying is delayed. Because gastric acids are decreased, gastric pH increases

You promote hemodynamic stability in a patient with upper GI bleeding by: A. Encouraging oral fluid intake B. Monitoring central venous pressure C. Monitoring laboratory test results and vital signs D. Giving blood, electrolyte and fluid replacement

D. Giving blood, electrolyte and fluid replacement Explanation: To stabilize a patient with acute bleeding, NS or LR solution is given I.V. until BP rises and urine output returns to 30ml/hr.

After an abdominal resection for colon cancer, Madeline returns to her room with a Jackson-Pratt drain in place. The purpose of the drain is to: A. Irrigate the incision with a saline solution B. Prevent bacterial infection of the incision C. Measure the amount of fluid lost after surgery D. Prevent accumulation of drainage in the wound

D. Prevent accumulation of drainage in the wound Explanation: A Jackson-Pratt drain promotes wound healing by allowing fluid to escape from the wound.

A client is prescribed with Omeprazole (Prilosec). The nurse determines that the client is receiving its therapeutic effect if which of the following is stated by the client: A. Relief of nausea and vomiting. B. Decrease diarrheal episodes. C. The absence of constipation. D. Relief from GERD.

D. Relief from GERD. Explanation: Omeprazole is used to treat symptoms of gastroesophageal reflux disease (GERD) and other conditions caused by excess stomach acid. It is also used to promote healing of erosive esophagitis (damage to your esophagus caused by stomach acid). Options B, C, and D are not related to this medication.

A client being treated for chronic cholecystitis should be given which of the following instructions? A. Increase rest B. Avoid antacids C. Increase protein in diet D. Use anticholinergics as prescribed

D. Use anticholinergics as prescribed Explanation: Conservative therapy for chronic cholecystitis includes weight reduction by increasing physical activity, a low-fat diet, antacid use to treat dyspepsia, and anticholinergic use to relax smooth muscles and reduce ductal tone and spasm, thereby reducing pain.

The nurse has provided dietary instructions to a client with a diagnosis of peptic ulcer disease. Which statement, if made by the client, indicates an understanding of the dietary measures to take? 1. "Baked foods such as chicken or fish are all right to eat." 2. "Citrus fruits and raw vegetables need to be included in my daily diet." 3. "I can drink beer so long as I consume only a moderate amount each day." 4. "I can drink coffee or tea so long as I limit the amount to two cups daily."

1. "Baked foods such as chicken or fish are all right to eat." Rationale: Dietary modifications for the client with peptic ulcer disease include eliminating foods that can cause irritation to the gastrointestinal (GI) tract. Items that should be eliminated or avoided include highly spiced foods, alcohol, caffeine, chocolate, and citrus fruits. Other foods may be taken according to the client's level of tolerance for that food.

The nurse is performing an assessment on a client with acute pancreatitis who was admitted to the hospital. Which assessment question would most specifically elicit information regarding the pain that is associated with acute pancreatitis? 1. "Does the pain in your stomach radiate to the back?" 2. "Does the pain in your lower abdomen radiate to the hip?" 3. "Does the pain in your lower abdomen radiate to your groin?" 4. "Does the pain in your stomach radiate to your lower middle abdomen?"

1. "Does the pain in your stomach radiate to the back?" Rationale: The pain that is associated with acute pancreatitis is often severe and is located in the epigastric region and radiates to the back. IGGY

A client experiencing chronic dumping syndrome makes the following comments to the nurse. Which one indicates the need for further teaching? 1. "I eat at least three large meals each day." 2. "I eat while lying in a semirecumbent position." 3. "I have eliminated taking liquids with my meals." 4. "I eat a high-protein, low- to moderate-carbohydrate diet."

1. "I eat at least three large meals each day." Rationale: Dumping syndrome describes a group of symptoms that occur after eating. It is believed to result from rapid dumping of gastric contents into the small intestine, which causes fluid to shift into the intestine. Signs and symptoms of dumping syndrome include diarrhea, abdominal distention, sweating, pallor, palpitations, and syncope. To manage this syndrome, clients are encouraged to decrease the amount of food taken at each sitting, eat in a semirecumbent position, eliminate ingesting fluids with meals, and avoid consumption of high-carbohydrate meals.

Sulfasalazine (Azulfidine) is prescribed for a client with a diagnosis of ulcerative colitis, and the care unit nurse instructs the client about the medication. Which statement made by the client indicates a need for further instruction? 1. "The medication will cause constipation." 2. "I need to take the medication with meals." 3. "I may have increased sensitivity to sunlight." 4. "This medication should be taken as prescribed."

1. "The medication will cause constipation." Rationale: Sulfasalazine is an anti-inflammatory sulfonamide. Constipation is not associated with this medication. It can cause photosensitivity, and the client should be instructed to avoid sun and ultraviolet light. It should be administered with meals, if possible, to prolong intestinal passage. The client needs to take the medication as prescribed and continue the full course of treatment even if symptoms are relieved.

The nurse is reviewing the prescription for a client admitted to the hospital with a diagnosis of acute pancreatitis. Which interventions would the nurse expect to be prescribed for the client? Select all that apply. 1. Administer antacids as prescribed. 2. Encourage coughing and deep breathing. 3. Administer anticholinergics as prescribed. 4. Give small, frequent high-calorie feedings. 5. Maintain the client in a supine and flat position. 6. Give opioid analgesics as prescribed for pain.

1. Administer antacids as prescribed. 2. Encourage coughing and deep breathing. 3. Administer anticholinergics as prescribed. 6. Give opioid analgesics as prescribed for pain. Rationale: The client with acute pancreatitis normally is placed on NPO status to rest the pancreas and suppress gastrointestinal secretions. Because abdominal pain is a prominent symptom of pancreatitis, pain medication is prescribed. Some clients experience lessened pain by assuming positions that flex the trunk, with the knees drawn up to the chest. A side-lying position with the head elevated 45 degrees decreases tension on the abdomen and may help ease the pain. The client is susceptible to respiratory infections because the retroperitoneal fluid raises the diaphragm, which causes the client to take shallow, guarded abdominal breaths. Therefore, measures such as turning, coughing, and deep breathing are instituted. Antacids and anticholinergics may be prescribed to suppress gastrointestinal secretions. IGGY

A client has just had a hemorrhoidectomy. Which nursing interventions are appropriate for this client? Select all that apply. 1. Administer stool softeners as prescribed. 2. Instruct the client to limit fluid intake to avoid urinary retention. 3. Instruct the client to avoid activities that will initiate vasovagal responses. 4. Encourage a high-fiber diet to promote bowel movements without straining. 5. Apply cold packs to the anal-rectal area over the dressing until the packing is removed. 6. Help the client to a Fowler's position to place pressure on the rectal area and decrease bleeding.

1. Administer stool softeners as prescribed. 4. Encourage a high-fiber diet to promote bowel movements without straining. 5. Apply cold packs to the anal-rectal area over the dressing until the packing is removed. Rationale: Nursing interventions after a hemorrhoidectomy are aimed at management of pain and avoidance of bleeding and incision rupture. Stool softeners and a high-fiber diet will help the client avoid straining, thereby reducing the chances of rupturing the incision. An ice pack will increase comfort and decrease bleeding. Options 2, 3, and 6 are incorrect interventions. IGGY

The nurse is reviewing the record of a client admitted to the nursing unit and notes that the client has a history of Laennec's cirrhosis. This type of cirrhosis is most commonly caused by which long-term condition? 1. Alcohol abuse 2. Cardiac disease 3. Exposure to chemicals 4. Obstruction to biliary ducts

1. Alcohol abuse Rationale: Laennec's cirrhosis results from long-term alcohol abuse. Cardiac cirrhosis most commonly is caused by long-term right-sided heart failure. Exposure to hepatotoxins, chemicals, or infections, or a metabolic disorder can cause postnecrotic cirrhosis. Biliary cirrhosis results from a decrease in bile flow and is most commonly caused by long-term obstruction of bile ducts. IGGY

The nurse notes that a client's parenteral nutrition solution is 4 hours behind. Which action should the nurse take? 1. Assess the infusion pump to be sure it is functioning properly and is set at the correct rate. 2. Increase the infusion rate to a rate that allows the infusion volume to correct itself within a 2-hour period. 3. Replace the parenteral nutrition solution with 10% dextrose, and restart the solution the following day. 4. Administer the parenteral nutrition solution using gravity flow because the infusion pump is malfunctioning.

1. Assess the infusion pump to be sure it is functioning properly and is set at the correct rate. Rationale: If an infusion falls behind schedule, the nurse should not increase the rate in an attempt to catch up, because a hyperosmotic reaction, among other reactions, could result. The solution should not be replaced by another or restarted the next day. An infusion pump should always be used to administer parenteral nutrition solution. IGGY

A client presents to the emergency department with upper gastrointestinal bleeding and is in moderate distress. In planning care, what is the priority nursing action for this client? 1. Assessment of vital signs 2. Completion of abdominal examination 3. Insertion of the prescribed nasogastric tube 4. Thorough investigation of precipitating events

1. Assessment of vital signs Rationale: The priority nursing action is to assess the vital signs. This would indicate the amount of blood loss that has occurred and provide a baseline by which to monitor the progress of treatment. The client may be unable to provide subjective data until the immediate physical needs are met. Although an abdominal examination and an assessment of the precipitating events may be necessary, these actions are not the priority. Insertion of a nasogastric tube is not the priority; in addition, the vital signs should be checked before performing this procedure. IGGY

A health care provider has written a prescription for ranitidine (Zantac) 300 mg once daily. The client indicates understanding of use of this medication by stating that the prescribed dose is best taken at what time? 1. At bedtime 2. After lunch 3. With supper 4. Before breakfast

1. At bedtime Rationale: Ranitidine is a histamine2 (H2)-receptor antagonist and should be taken at bedtime, when it is given as a single daily dose. This allows for prolonged effect and provides the greatest protection of the gastric mucosa both during sleep and around the clock.

A Penrose drain is in place on the first postoperative day in a client who has undergone a cholecystectomy procedure. Serosanguineous drainage is noted on the dressing covering the drain. Which nursing intervention is most appropriate? 1. Change the dressing. 2. Continue to monitor the drainage. 3. Notify the health care provider (HCP). 4. Use a pen to circle the amount of drainage on the dressing.

1. Change the dressing. Rationale: Serosanguineous drainage with a small amount of bile is expected from the Penrose drain for the first 24 hours. Drainage then decreases, and the drain is usually removed within 48 hours. A sterile dressing covers the site and should be changed if wet to prevent infection and skin excoriation. Although the nurse would continue to monitor the drainage, the most appropriate intervention is to change the dressing. The HCP does not need to be notified. IGGY

The nurse is planning to teach a client with gastroesophageal reflux disease about substances to avoid. Which items should the nurse include on this list? Select all that apply. 1. Coffee 2. Chocolate 3. Peppermint 4. Nonfat milk 5. Fried chicken 6. Scrambled eggs

1. Coffee 2. Chocolate 3. Peppermint 5. Fried chicken Rationale: Foods that decrease lower esophageal sphincter (LES) pressure and irritate the esophagus will increase reflux and exacerbate the symptoms of gastroesophageal reflux disease (GERD) and therefore should be avoided. Aggravating substances include chocolate, coffee, fried or fatty foods, peppermint, carbonated beverages, and alcohol. Options 4 and 6 do not promote this effect. IGGY

The nurse inspects the color of the drainage from a nasogastric tube on a postoperative client approximately 24 hours after gastric surgery. Which finding indicates the need to notify the health care provider? 1. Dark red drainage 2. Dark brown drainage 3. Green-tinged drainage 4. Light yellowish brown drainage

1. Dark red drainage Rationale: For the first 12 hours after gastric surgery, the nasogastric tube drainage may be dark brown to dark red. Later, the drainage should change to a light yellowish brown color. The presence of bile may cause a green tinge. The health care provider (HCP) should be notified if dark red drainage, a sign of hemorrhage, is noted 24 hours postoperatively. IGGY

The nurse is reviewing the record of a client with Crohn's disease. Which stool characteristic should the nurse expect to note documented in the client's record? 1. Diarrhea 2. Chronic constipation 3. Constipation alternating with diarrhea 4. Stool constantly oozing from the rectum

1. Diarrhea Rationale: Crohn's disease is characterized by nonbloody diarrhea of usually not more than four to five stools daily. Over time, the diarrhea episodes increase in frequency, duration, and severity. IGGY

The nurse is reviewing the results of serum laboratory studies for a client admitted for suspected hepatitis. Which laboratory finding is most associated with hepatitis requiring the nurse to contact the health care provider? 1. Elevated serum bilirubin level 2. Below normal hemoglobin concentration 3. Elevated blood urea nitrogen (BUN) level 4. Elevated erythrocyte sedimentation rate (ESR)

1. Elevated serum bilirubin level Rationale: Laboratory indicators of hepatitis include elevated liver enzymes, serum bilirubin level, and ESR. However, ESR is a nonspecific test that indicates the presence of inflammation somewhere in the body. The hemoglobin concentration is unrelated to this diagnosis. An elevated BUN level may indicate renal dysfunction. IGGY

The nurse has been caring for a client who required a Sengstaken-Blakemore tube because other treatment measures for esophageal varices were unsuccessful. The health care provider arrives on the nursing unit and deflates the esophageal balloon. After deflation of the balloon, the nurse should monitor the client most closely for which complication? 1. Hematemesis 2. Bloody diarrhea 3. Swelling of the abdomen 4. An elevated temperature and a rise in blood pressure

1. Hematemesis Rationale: A Sengstaken-Blakemore tube may be inserted in a client with a diagnosis of cirrhosis and ruptured esophageal varices. It has both an esophageal and a gastric balloon. The esophageal balloon exerts pressure on the ruptured esophageal varices and stops the bleeding. The pressure of the esophageal balloon is released at intervals to decrease the risk of trauma to esophageal tissues, including esophageal rupture or necrosis. When the balloon is deflated, the client may begin to bleed again from the esophageal varices, manifested as vomiting of blood (hematemesis).

The health care provider has determined that a client with hepatitis has contracted the infection from contaminated food. The nurse understands that this client is most likely experiencing what type of hepatitis? 1. Hepatitis A 2. Hepatitis B 3. Hepatitis C 4. Hepatitis D

1. Hepatitis A Rationale: Hepatitis A is transmitted by the fecal-oral route via contaminated food or infected food handlers. Hepatitis B, C, and D are transmitted most commonly via infected blood or body fluids IGGY

The nurse is caring for a client with acute pancreatitis and is monitoring the client for paralytic ileus. Which assessment data should alert the nurse to this occurrence? 1. Inability to pass flatus 2. Loss of anal sphincter control 3. Severe, constant pain with rapid onset 4. Firm, nontender mass palpable at the lower right costal margin

1. Inability to pass flatus Rationale: An inflammatory reaction such as acute pancreatitis can cause paralytic ileus, the most common form of nonmechanical obstruction. Inability to pass flatus is a clinical manifestation of paralytic ileus. Loss of sphincter control is not a sign of paralytic ileus. Pain is associated with paralytic ileus, but the pain usually manifests as a more constant generalized discomfort. Option 4 is the description of the physical finding of liver enlargement. The liver may be enlarged in cases of cirrhosis or hepatitis. Although this client may have an enlarged liver, an enlarged liver is not a sign of paralytic ileus or intestinal obstruction. IGGY

The nurse is teaching the client with viral hepatitis about the stages of the disease. The nurse should explain to the client that the second stage of this disease is characterized by which specific assessment findings? Select all that apply. 1. Jaundice 2. Flu-like symptoms 3. Clay-colored stools 4. Dark or tea-colored urine 5. Elevated bilirubin levels

1. Jaundice 3. Clay-colored stools 4. Dark or tea-colored urine 5. Elevated bilirubin levels Rationale: There are three stages associated with viral hepatitis. The first (preicteric) stage includes flulike symptoms only. The second (icteric) stage includes the appearance of jaundice and associated symptoms such as elevated bilirubin levels, dark or tea-colored urine, and clay-colored stools. The third (posticteric) stage occurs when the jaundice decreases and the colors of the urine and stool return to normal. IGGY

The nurse is giving dietary instructions to a client who has a new colostomy. The nurse should encourage the client to eat foods representing which diet for the first 4 to 6 weeks postoperatively? 1. Low fiber 2. Low calorie 3. High protein 4. High carbohydrate

1. Low fiber Rationale: For the first 4 to 6 weeks after colostomy formation, the client should consume a low-fiber diet. After this period, the client should eat a high-carbohydrate, high-protein diet. The client also is instructed to add new foods, including those with fiber, one at a time to determine tolerance to that food.

The nurse is reviewing the laboratory results for a client with cirrhosis and notes that the ammonia level is elevated. Which diet does the nurse anticipate to be prescribed for this client? 1. Low-protein diet 2. High-protein diet 3. Moderate-fat diet 4. High-carbohydrate diet

1. Low-protein diet Rationale: Cirrhosis is a chronic, progressive disease of the liver characterized by diffuse degeneration and destruction of hepatocytes. Most of the ammonia in the body is found in the gastrointestinal tract. Protein provided by the diet is transported to the liver by the portal vein. The liver breaks down protein, which results in the formation of ammonia. If the client has hepatic encephalopathy, a low-protein diet would be prescribed.

A client with hiatal hernia chronically experiences heartburn following meals. The nurse should plan to teach the client to avoid which action because it is contraindicated with a hiatal hernia? 1. Lying recumbent following meals 2. Consuming small, frequent, bland meals 3. Raising the head of the bed on 6-inch blocks 4. Taking H2-receptor antagonist medication

1. Lying recumbent following meals Rationale: Hiatal hernia is caused by a protrusion of a portion of the stomach above the diaphragm where the esophagus usually is positioned. The client usually experiences pain from reflux caused by ingestion of irritating foods, lying flat following meals or at night, and eating large or fatty meals. Relief is obtained with the intake of small, frequent, and bland meals, use of H2-receptor antagonists and antacids, and elevation of the thorax following meals and during sleep. IGGY

A client has developed hepatitis A after eating contaminated oysters. The nurse assesses the client for which expected assessment finding? 1. Malaise 2. Dark stools 3. Weight gain 4. Left upper quadrant discomfort

1. Malaise Rationale: Hepatitis causes gastrointestinal symptoms such as anorexia, nausea, right upper quadrant discomfort, and weight loss. Fatigue and malaise are common. Stools will be light- or clay-colored if conjugated bilirubin is unable to flow out of the liver because of inflammation or obstruction of the bile ducts. IGGY

A histamine (H2)-receptor antagonist will be prescribed for a client. The nurse understands that which medications are H2-receptor antagonists? Select all that apply. 1. Nizatidine (Axid) 2. Ranitidine (Zantac) 3. Famotidine (Pepcid) 4. Cimetidine (Tagamet) 5. Esomeprazole (Nexium) 6. Lansoprazole (Prevacid)

1. Nizatidine (Axid) 2. Ranitidine (Zantac) 3. Famotidine (Pepcid) 4. Cimetidine (Tagamet) Rationale: H2-receptor antagonists suppress secretion of gastric acid, alleviate symptoms of heartburn, and assist in preventing complications of peptic ulcer disease. These medications also suppress gastric acid secretions and are used in active ulcer disease, erosive esophagitis, and pathological hypersecretory conditions. The other medications listed are proton pump inhibitors. Kee Hayes

A client with cirrhosis has ascites and excess fluid volume. Which assessment findings does the nurse anticipate to note as a result of increased abdominal pressure? Select all that apply. 1. Orthopnea, dyspnea 2. Petechiae and ecchymosis 3. Inguinal or umbilical hernia 4. Poor body posture and balance 5. Abdominal distention and tenderness

1. Orthopnea, dyspnea 2. Petechiae and ecchymosis 3. Inguinal or umbilical hernia 5. Abdominal distention and tenderness Rationale: Excess fluid volume, related to the accumulation of fluid in the peritoneal cavity and dependent areas of the body, can occur in the client with cirrhosis. Ascites can cause physical problems because of the overdistended abdomen and resultant pressure on internal organs and vessels. These problems include respiratory difficulty, petechiae and ecchymoses, development of hernias, abdominal distention, and tenderness. IGGY

A client has been advanced to a solid diet after undergoing a subtotal gastrectomy. The nurse caring for the client would perform which action to minimize the risk of dumping syndrome? 1. Remove fluids from the meal tray. 2. Give the client two large meals per day. 3. Ask the client to sit up for 1 hour after eating. 4. Provide concentrated, high-carbohydrate foods.

1. Remove fluids from the meal tray. Rationale: Factors to minimize dumping syndrome after gastric surgery include having the client lie down for at least 30 minutes after eating; giving small, frequent meals; having the client maintain a low Fowler's position while eating if possible; avoiding liquids with meals; and avoiding high-carbohydrate food sources. Antispasmodic medications also are prescribed as needed to delay gastric emptying.

A sexually active 20-year-old client has developed viral hepatitis. Which client statement indicates the need for further teaching? 1. "I should avoid drinking alcohol." 2. "I can go back to work right away." 3. "My partner should get the vaccine." 4. "A condom should be used for sexual intercourse."

2. "I can go back to work right away." Rationale: To prevent transmission of hepatitis, vaccination of the partner is advised. In addition, a condom is advised during sexual intercourse. Alcohol should be avoided because it is detoxified in the liver and may interfere with recovery. Rest is especially important until laboratory studies show that liver function has returned to normal. The client's activity is increased gradually, and the client should not return to work right away. IGGY

Dicyclomine hydrochloride (Bentyl) has been prescribed for a client with irritable bowel syndrome, and the nurse provides instructions to the client about how to take this medication. Which statement, if made by the client, best indicates an understanding of how to take this medication? 1. "I should take the pill with food and at meal times." 2. "I should take the pill 30 minutes before each meal." 3. "I should take the pill after I have finished eating my meal." 4. "I should take the pill when I first wake up in the morning and right before I go to bed."

2. "I should take the pill 30 minutes before each meal." Rationale: Dicyclomine hydrochloride is an anticholinergic, antispasmodic agent often used to treat irritable bowel syndrome that is unresponsive to diet therapy. To be effective in decreasing bowel motility, antispasmodic medication should be administered 30 minutes before meals.

An ultrasound examination of the gallbladder is scheduled for a client with a suspected diagnosis of cholecystitis. Correct instructions about the procedure should include which statement? 1. "This procedure may cause discomfort." 2. "This test requires that you lie still for short intervals." 3. "This procedure is preceded by the administration of oral tablets." 4. "This procedure requires that you not eat or drink anything for 24 hours before the test."

2. "This test requires that you lie still for short intervals." Rationale: Ultrasound examination of the gallbladder is a noninvasive procedure and frequently is used for emergency diagnosis of acute cholecystitis. It is a painless test and does not require the administration of oral tablets as preparation. The client does not need to be NPO for 24 hours before the test but may be instructed to avoid carbonated beverages for 48 hours before the test to help decrease intestinal gas. IGGY

The ambulatory care nurse is providing instructions to a client who is scheduled for a colonoscopy to remove a polyp. Which instructions are appropriate for client preparation for this procedure? 1. Clear liquids may be consumed starting 24 hours after the procedure. 2. A bowel preparation will be required in preparation for the procedure. 3. Clear liquids only are allowed on the day of the scheduled procedure. 4. If blood-tinged stools are noted after the procedure, the health care provider (HCP) should be notified.

2. A bowel preparation will be required in preparation for the procedure. Rationale: The client should be instructed that bowel preparation with a laxative is prescribed before the procedure to cleanse the bowel. Oral intake is allowed after the procedure once the client is stable. A clear liquid diet is permitted on the day before the procedure (per HCP preference), and then oral intake is avoided for 8 hours immediately before the procedure. If a polyp has been removed, the client is instructed that the stool may be tinged with blood. However, any signs of tenderness, abdominal pain, or bloody stools should be reported to the HCP.

The nurse is caring for a client who is receiving intermittent feeding via a nasogastric (NG) tube. Before administering a feeding to the client, the nurse should perform which action first? 1. Warm the feeding to 103° F. 2. Check the placement of the tube. 3. Rinse the Asepto syringe with warm water. 4. Check the last time medications were given.

2. Check the placement of the tube. Rationale: To prevent aspiration while administering a tube feeding, the nurse should place the client in an upright sitting position or elevate the head of the bed at least 30 degrees. Before the feeding, the nurse checks the placement of the tube by aspirating gastric contents and measuring the pH. If the tube is not in the stomach, the client is at risk for aspiration. Formulas are administered at room temperature. Rinsing the Asepto syringe with warm water and checking the last time the medication was administered are not directly related to the information in the question. IGGY

A client with Crohn's disease is scheduled to receive an infusion of infliximab (Remicade). What intervention by the nurse will determine the effectiveness of treatment? 1. Monitoring the leukocyte count for 2 days after the infusion 2. Checking the frequency and consistency of bowel movements 3. Checking serum liver enzyme levels before and after the infusion 4. Carrying out a Hematest on gastric fluids after the infusion is completed

2. Checking the frequency and consistency of bowel movements Rationale: The principal manifestations of Crohn's disease are diarrhea and abdominal pain. Infliximab (Remicade) is an immunomodulator that reduces the degree of inflammation in the colon, thereby reducing the diarrhea.

The nurse is providing dietary instructions to a client with a diagnosis of irritable bowel syndrome. The nurse determines that the client understands the instructions if the client states the need to avoid which food? 1. Rice 2. Corn 3. Broiled chicken 4. Cream of wheat

2. Corn Rationale: The client with irritable bowel should take in a diet that consists of 30g to 40 g of fiber daily because dietary fiber will help produce bulky soft stools and establish regular bowel habits. The client should also drink 8 to 10 glasses of fluid daily and chew food slowly to promote normal bowel function. Foods that are irritating to the intestines need to be avoided. Corn is high in fiber but can be very irritating to the intestines and should be avoided. The food items in the other options are acceptable to eat.

A nurse is caring for a hospitalized client who has been diagnosed with pancreatitis. The nurse checks the laboratory results form, anticipating that which enzyme will remain normal in the client? 1. Lipase 2. Lactase 3. Trypsin 4. Amylase

2. Lactase Rationale: Lactase is produced in the small intestine and aids in splitting neutral fats into glycerol and fatty acids. Lipase, trypsin, and amylase are produced in the pancreas and aid in the digestion of fats, starches, and proteins, respectively. IGGY

The nurse is preparing to administer prescribed medications to a client with hepatic encephalopathy. The nurse anticipates that the health care provider's prescriptions will include which medication? 1. Bisacodyl (Dulcolax) 2. Lactulose (Chronulac) 3. Magnesium hydroxide (Milk of Magnesia) 4. Psyllium hydrophilic mucilloid (Metamucil)

2. Lactulose (Chronulac) Rationale: Lactulose is a hyperosmotic laxative agent that has the adjunct benefit of lowering serum ammonia levels. This occurs because the medication lowers bowel pH and aids in the conversion of ammonia in the gut to the ammonium ion, which is poorly absorbed. Bisacodyl is a stimulant laxative. Magnesium hydroxide is a saline laxative. Psyllium hydrophilic mucilloid is a bulk laxative.

The nurse is preparing to insert a nasogastric (NG) tube as prescribed for the purpose of stomach decompression. The nurse reviews the health care provider's (HCP) prescriptions and anticipates that the HCP will prescribe which type of suction pressure and control? 1. High and intermittent 2. Low and intermittent 3. High and continuous 4. Low and continuous

2. Low and intermittent Rationale: Gastric mucosa can be traumatized and pulled into the tube if the suction pressure is placed on high or if the suction is continuous. The suction should be set on low pressure and intermittent suction control. IGGY

A client who chronically uses nonsteroidal antiinflammatory drugs (NSAIDs) has been taking misoprostol (Cytotec). The nurse determines that the medication is having the intended therapeutic effect if which finding is noted? 1. Resolved diarrhea 2. Relief of epigastric pain 3. Decreased platelet count 4. Decreased white blood cell count

2. Relief of epigastric pain Rationale: The client who chronically uses nonsteroidal antiinflammatory drugs (NSAIDs) is prone to gastric mucosal injury. Misoprostol is a gastric protectant and is given specifically to prevent this occurrence. Diarrhea can be a side effect of the medication but is not an intended effect. Options 3 and 4 are incorrect.

A client has been admitted to the hospital with a diagnosis of acute pancreatitis and the nurse is assessing the client's pain. What type of pain is consistent with this diagnosis? 1. Burning and aching, located in the left lower quadrant and radiating to the hip 2. Severe and unrelenting, located in the epigastric area and radiating to the back 3. Burning and aching, located in the epigastric area and radiating to the umbilicus 4. Severe and unrelenting, located in the left lower quadrant and radiating to the groin

2. Severe and unrelenting, located in the epigastric area and radiating to the back Rationale: The pain associated with acute pancreatitis is often severe and unrelenting, is located in the epigastric region, and radiates to the back. The other options are incorrect. IGGY

A health care provider is about to perform a paracentesis for a client with abdominal ascites. The nurse assisting with the procedure should help the client into which position? 1. Supine 2. Upright 3. Right side-lying 4. Left side-lying

2. Upright Rationale: An upright position allows the intestine to float posteriorly and helps prevent intestinal laceration during catheter insertion. The client ideally sits upright in a chair, with the feet flat on the floor and with the bladder emptied before the procedure. IGGY

The nurse is obtaining a health history for a client with chronic pancreatitis. The health history is most likely to include which as a common causative factor in this client's disorder? 1. Weight gain 2. Use of alcohol 3. Exposure to occupational chemicals 4. Abdominal pain relieved with food or antacids

2. Use of alcohol Rationale: Chronic pancreatitis occurs most often in alcoholics. Abstinence from alcohol is important to prevent the client from developing chronic pancreatitis. Clients usually experience malabsorption with weight loss. Chemical exposure is associated with cancer of the pancreas. Pain will not be relieved with food or antacids. IGGY

The nurse is caring for a client with a diagnosis of chronic gastritis. The nurse monitors the client, knowing that this client is at risk for which vitamin deficiency? 1. Vitamin A 2. Vitamin B12 3. Vitamin C 4. Vitamin E

2. Vitamin B12 Rationale: Chronic gastritis causes deterioration and atrophy of the lining of the stomach, leading to the loss of function of the parietal cells. The source of intrinsic factor is lost, which results in an inability to absorb vitamin B12. This leads to the development of pernicious anemia. IGGY

The nurse is caring for a client who had a subtotal gastrectomy. The nurse should assess the client for which signs and symptoms of dumping syndrome? 1. Diarrhea, chills, and hiccups 2. Weakness, diaphoresis, and diarrhea 3. Fever, constipation, and rectal bleeding 4. Abdominal pain, elevated temperature, and weakness

2. Weakness, diaphoresis, and diarrhea Rationale: Dumping syndrome occurs after gastric surgery because food is not held long enough in the stomach and is "dumped" into the small intestine as a hypertonic mass. This causes fluid to shift into the intestines, causing cardiovascular and gastrointestinal symptoms. Signs and symptoms typically include weakness, dizziness, diaphoresis, flushing, hypotension, abdominal pain and distention, hyperactive bowel sounds, and diarrhea. IGGY

An older client recently has been taking cimetidine (Tagamet). The nurse monitors the client for which most frequent central nervous system side effect of this medication? 1. Tremors 2. Dizziness 3. Confusion 4. Hallucinations

3. Confusion Rationale: Cimetidine is a histamine (H2)-receptor antagonist. Older clients are especially susceptible to central nervous system side effects of cimetidine. The most frequent of these is confusion. Less common central nervous system side effects include headache, dizziness, drowsiness, and hallucinations

The nurse has given instructions to a client with hepatitis about post-discharge management during convalescence. The nurse determines that further teaching is needed if the client makes which statement? 1. "I need to avoid alcohol and aspirin." 2. "I should eat a high-carbohydrate, low-fat diet." 3. "I can resume a full activity level within 1 week." 4. "I need to take the prescribed amounts of vitamin K."

3. "I can resume a full activity level within 1 week." Rationale: The client with hepatitis is easily fatigued and may require several weeks to resume a full activity level. It is important for the client to get adequate rest so that the liver can heal. The client should avoid hepatotoxic substances such as aspirin and alcohol. The client should take in a high-carbohydrate and low-fat diet. Vitamin K may be prescribed for prolonged clotting times.

The nurse is doing preoperative teaching with a client who is about to undergo creation of a Kock pouch. The nurse interprets that the client has the best understanding of the nature of the surgery if the client makes which statement? 1. "I will be able to pass stool by the rectum eventually." 2. "The drainage from this type of ostomy will be formed." 3. "I will need to drain the pouch regularly with a catheter." 4. "I will need to wear a drainage bag for the rest of my life."

3. "I will need to drain the pouch regularly with a catheter." Rationale: A Kock pouch is a continent ileostomy. As the ileostomy begins to function, the client drains it every 3 to 4 hours and then decreases the draining to about three times a day, or as needed when full. The client does not need to wear a drainage bag but should wear an absorbent dressing to absorb mucous drainage from the stoma. IGGY

A client with a peptic ulcer is diagnosed with a Helicobacter pylori infection. The nurse is teaching the client about the medications prescribed, including clarithromycin (Biaxin), esomeprazole (Nexium), and amoxicillin (Amoxil). Which statement by the client indicates the best understanding of the medication regimen? 1. "My ulcer will heal because these medications will kill the bacteria." 2. "These medications are only taken when I have pain from my ulcer." 3. "The medications will kill the bacteria and stop the acid production." 4. "These medications will coat the ulcer and decrease the acid production in my stomach."

3. "The medications will kill the bacteria and stop the acid production." Rationale: Triple therapy for Helicobacter pylori infection usually includes two antibacterial medications and a proton pump inhibitor. Clarithromycin and amoxicillin are antibacterials. Esomeprazole is a proton pump inhibitor. These medications will kill the bacteria and decrease acid production. IGGY

A client seen in the ambulatory care clinic has ascites and slight jaundice. The nurse should assess the client for a history of chronic use of which medication? 1. Ibuprofen (Advil) 2. Ranitidine (Zantac) 3. Acetaminophen (Tylenol) 4. Acetylsalicylic acid (aspirin)

3. Acetaminophen (Tylenol) Rationale: Acetaminophen is a potentially hepatotoxic medication. Use of this medication and other hepatotoxic agents should be investigated whenever a client presents with signs and symptoms compatible with liver disease (such as ascites and jaundice).

The nurse is developing a teaching plan for a client with viral hepatitis. The nurse should plan to include which information in the teaching session? 1. The diet should be low in calories. 2. Meals should be large to conserve energy. 3. Activity should be limited to prevent fatigue. 4. Alcohol intake should be limited to 2 ounces per day.

3. Activity should be limited to prevent fatigue. Rationale: Rest is necessary for the client with hepatitis, and the client with viral hepatitis should limit activity to avoid fatigue. The diet should be optimal in calories, proteins, and carbohydrates. The client should take in several small meals per day. Alcohol is strictly forbidden.

A client with liver dysfunction is having difficulty with protein metabolism. The nurse checks the laboratory results, expecting that the results of which serum laboratory values will be elevated? 1. Lactase 2. Albumin 3. Ammonia 4. Lactic acid

3. Ammonia Rationale: During deamination of proteins in the liver, the amino group splits from the carbon-containing compound, which results in formation of ammonia and a carbon residue. The liver then converts the toxic ammonia substance into urea, which can be excreted by the kidneys. Clients with liver dysfunction may have high serum ammonia levels as a result. Options 1, 2, and 4 are incorrect. IGGY

A hospitalized client is diagnosed with pancreatitis. The nurse plans care, knowing that production of which substance will be elevated in blood studies for this client? 1. Pepsin 2. Lactase 3. Amylase 4. Enterokinase

3. Amylase Rationale: The serum level of amylase, an enzyme produced by the pancreas, increases with pancreatitis. Amylase normally is responsible for carbohydrate digestion. Pepsin is produced by the stomach and is used in protein digestion. Lactase and enterokinase are enzymes produced by the small intestine; lactase splits lactose into galactose and fructose, and enterokinase activates trypsin. IGGY

A client has a PRN prescription for loperamide hydrochloride (Imodium). For which condition should the nurse administer this medication? 1. Constipation 2. Abdominal pain 3. An episode of diarrhea 4. Hematest-positive nasogastric tube drainage

3. An episode of diarrhea Rationale: Loperamide is an antidiarrheal agent. It is used to manage acute and chronic diarrhea in conditions such as inflammatory bowel disease. Loperamide also can be used to reduce the volume of drainage from an ileostomy.

The nurse is reviewing the record of a client with a diagnosis of cirrhosis and notes that there is documentation of the presence of asterixis. How should the nurse assess for its presence? 1. Dorsiflex the client's foot. 2. Measure the abdominal girth. 3. Ask the client to extend the arms. 4. Instruct the client to lean forward.

3. Ask the client to extend the arms. Rationale: Asterixis is irregular flapping movements of the fingers and wrists when the hands and arms are outstretched, with the palms down, wrists bent up, and fingers spread. Asterixis is the most common and reliable sign that hepatic encephalopathy is developing.

The nurse is developing a plan of care for a client who is receiving total parenteral nutrition (TPN). The nurse identifies assessments to be made to help identify complications related to the infusion of the TPN solution. The care plan should include monitoring of which assessment item(s)? 1. Apical rate 2. Pulse oximetry 3. Blood glucose levels 4. Hemoglobin and hematocrit

3. Blood glucose levels Rationale: Complications associated with TPN therapy include hypoglycemia or hyperglycemia, infection, fluid overload, air embolism, and electrolyte imbalance. It is standard care to monitor blood glucose levels at 6-hour intervals to assess for hyperglycemia. IGGY

The nurse has administered prochlorperazine to a client for relief of nausea and vomiting. The nurse should then assess the client for which frequent side effect of this medication? 1. Diarrhea 2. Drooling 3. Blurred vision 4. Excessive tearing

3. Blurred vision Rationale: Prochlorperazine is a phenothiazine-type antiemetic and antipsychotic agent. A frequent side effect is blurred vision. Other frequent side effects of this medication are constipation, dry mouth, and dry eyes.

A nurse is developing a plan of care for a client who has undergone an esophagogastroduodenoscopy (EGD) procedure. The nurse should include which intervention in the nursing care plan? 1. Monitor the client's vital signs every hour for 4 hours. 2. Place the client in a prone position to provide comfort. 3. Check the gag reflex by using a tongue depressor to stroke the back of the client's throat. 4. Provide saline gargles immediately on the client's return to the nursing unit to aid in comfort.

3. Check the gag reflex by using a tongue depressor to stroke the back of the client's throat. Rationale: After EGD, the vital signs are checked frequently, usually every 30 minutes, until sedation wears off. The nurse places the client in a semi-Fowler's to Fowler's position to aid in comfort and prevent aspiration. The client remains on NPO (nothing by mouth) status until the gag reflex returns (usually in 2 to 4 hours). The nurse can check the gag reflex by using a tongue depressor to stroke the back of client's throat. Saline gargles would not be allowed until the gag reflex returns.

The nurse instructs a client taking aluminum hydroxide (Amphojel) that the most common side effect associated with administration of this medication is which effect? 1. Diarrhea 2. Headache 3. Constipation 4. Muscle weakness

3. Constipation Rationale: Aluminum-containing antacids are constipating, so the client should be instructed to take a stool softener or additional bulk-type laxatives to relieve this uncomfortable side effect. Options 1, 2, and 4 are not side effects of this medication. Kee Hayes

Lactulose (Chronulac) is prescribed for a client with a diagnosis of hepatic encephalopathy. The nurse should determine that this medication has had a therapeutic effect if assessment reveals which finding? 1. Increased protein level 2. Increased red blood cell count 3. Decreased serum ammonia level 4. Decreased white blood cell count

3. Decreased serum ammonia level Rationale: Lactulose is prescribed for the client with hepatic encephalopathy to reduce bacterial breakdown of protein in the bowel. The medication creates an acidic environment in the bowel and causes the ammonia to leave the bloodstream and enter the colon. Ammonia then becomes trapped in the bowel. Lactulose also has a laxative effect that allows for the elimination of the ammonia.

The nurse has provided home care instructions to a client who had a subtotal gastrectomy. The nurse instructs the client regarding the signs and symptoms associated with dumping syndrome. Which signs and symptoms, if identified by the client, would indicate an understanding of this potential complication after gastrointestinal (GI) surgery? 1. Hiccups and diarrhea 2. Constipation and fever 3. Diaphoresis and diarrhea 4. Fatigue and abdominal pain

3. Diaphoresis and diarrhea Rationale: Dumping syndrome occurs after gastric surgery because food is not held as long in the stomach and is dumped into the intestine as a hypertonic mass. This causes fluid to shift into the intestine, causing cardiovascular and GI symptoms. Symptoms can typically include weakness, dizziness, diaphoresis, flushing, hypotension, abdominal pain and distention, hyperactive bowel sounds, and diarrhea.

The nurse is assessing a client 24 hours following a cholecystectomy. The nurse notes that the T-tube has drained 750 mL of green-brown drainage since the surgery. Which nursing intervention is most appropriate? 1. Clamp the T-tube. 2. Irrigate the T-tube. 3. Document the findings. 4. Notify the health care provider.

3. Document the findings Rationale: Following cholecystectomy, drainage from the T-tube is initially bloody and then turns a greenish-brown color. The drainage is measured as output. The amount of expected drainage will range from 500 to 1000 mL/day. The nurse would document the output. IGGY

The nurse is monitoring the status of a client's fat emulsion (lipid) infusion and notes that the infusion is 1 hour behind. Which action should the nurse take? 1. Adjust the infusion rate to catch up over the next hour. 2. Increase the infusion rate to catch up over the next 2 hours. 3. Ensure that the fat emulsion infusion rate is infusing at the prescribed rate. 4. Adjust the infusion rate to run wide open until the solution is back on time.

3. Ensure that the fat emulsion infusion rate is infusing at the prescribed rate. Rationale: The nurse should not increase the rate of a fat emulsion to make up the difference if the infusion timing falls behind. Doing so could place the client at risk for fat overload. In addition, increasing the rate suddenly can cause fluid overload. The same principle (not increasing the rate) applies to PN or any intravenous (IV) infusion. Therefore the remaining options are incorrect.

The nurse is scheduling diagnostic tests for a client. Which of the diagnostic tests prescribed should be performed last? 1. Barium enema 2. Barium swallow 3. Gallbladder series 4. Oral cholecystogram

3. Gallbladder series Rationale: A barium swallow should be done after a barium enema or gallbladder series to prevent the contrast used in the barium swallow from obstructing the view of other organs. It takes several days for swallowed barium to pass completely out of the gastrointestinal tract. Pagana, Pagana

The nurse is caring for a client following a Billroth II procedure. Which postoperative prescription should the nurse question and verify? 1. Leg exercises 2. Early ambulation 3. Irrigating the nasogastric tube 4. Coughing and deep-breathing exercises

3. Irrigating the nasogastric tube Rationale: In a Billroth II procedure, the proximal remnant of the stomach is anastomosed to the proximal jejunum. Patency of the nasogastric tube is critical for preventing the retention of gastric secretions. The nurse should never irrigate or reposition the gastric tube after gastric surgery, unless specifically prescribed by the health care provider. In this situation, the nurse should clarify the prescription. Options 1, 2, and 4 are appropriate postoperative interventions. IGGY

The nurse is providing discharge instructions to a client following gastrectomy and should instruct the client to take which measure to assist in preventing dumping syndrome? 1. Ambulate following a meal. 2. Eat high-carbohydrate foods. 3. Limit the fluids taken with meals. 4. Sit in a high Fowler's position during meals.

3. Limit the fluids taken with meals Rationale: Dumping syndrome is a term that refers to a constellation of vasomotor symptoms that occurs after eating, especially following a Billroth II procedure. Early manifestations usually occur within 30 minutes of eating and include vertigo, tachycardia, syncope, sweating, pallor, palpitations, and the desire to lie down. The nurse should instruct the client to decrease the amount of fluid taken at meals and to avoid high-carbohydrate foods, including fluids such as fruit nectars; to assume a low Fowler's position during meals; to lie down for 30 minutes after eating to delay gastric emptying; and to take antispasmodics as prescribed.

A client has been diagnosed with gastroesophageal reflux disease (GERD). The nurse plans care, knowing that the client has dysfunction of which part of the digestive system? 1. Chief cells of the stomach 2. Parietal cells of the stomach 3. Lower esophageal sphincter (LES) 4. Upper esophageal sphincter (UES)

3. Lower esophageal sphincter (LES) Rationale: The LES is a functional sphincter that normally remains closed except when food or fluids are swallowed. If relaxation of this sphincter occurs, the client may experience symptoms of GERD. The chief cells of the stomach secrete pepsinogen, a precursor to pepsin, which helps to digest proteins. The parietal cells of the stomach secrete hydrochloric acid (gastric acid) and intrinsic factor. The UES is formed by the cricopharyngeus muscle attached to the cricoid cartilage. IGGY

The nurse is providing instructions to a client regarding measures to minimize the risk of dumping syndrome. The nurse should make which suggestion to the client? 1. Maintain a high-carbohydrate diet. 2. Increase fluid intake, particularly at meal time. 3. Maintain a low Fowler's position while eating. 4. Ambulate for at least 30 minutes following each meal.

3. Maintain a low Fowler's position while eating. Rationale: The client at risk for dumping syndrome should be instructed to maintain a low Fowler's position while eating and lie down for at least 30 minutes after eating. The client also should be told that small, frequent meals are best and to avoid liquids with meals. Avoiding high-carbohydrate food sources also will assist in minimizing dumping syndrome. IGGY

The nurse is assessing a client with a duodenal ulcer. The nurse interprets that which sign/symptom is most consistent with the typical presentation of duodenal ulcer? 1. Weight loss 2. Nausea and vomiting 3. Pain that is relieved by food intake 4. Pain that radiates down the right arm

3. Pain that is relieved by food intake Rationale: The most typical finding with duodenal ulcer is pain that is relieved by food intake. The pain is often described as a burning, heavy, sharp, or "hunger pang" pain that often localizes in the midepigastric area. It does not radiate down the right arm. The client with duodenal ulcer does not usually experience weight loss or nausea and vomiting; these symptoms are more typical in the client with a gastric ulcer. IGGY

A client is experienced delayed gastric emptying. The nurse plans care, knowing that dysfunction of which structures is responsible for the client's symptoms? 1. Ileum 2. Jejunum 3. Pyloric sphincter 4. Cardiac sphincter

3. Pyloric sphincter Rationale: The pyloric sphincter joins the stomach and the duodenum and is responsible for proper gastric emptying. The ileum and jejunum are the two portions of the small intestine. The cardiac sphincter is another name for the lower esophageal sphincter, which joins the esophagus and the stomach. IGGY

A client has begun medication therapy with pancrelipase (Pancrease MT). The nurse evaluates that the medication is having the optimal intended benefit if which effect is observed? 1. Weight loss 2. Relief of heartburn 3. Reduction of steatorrhea 4. Absence of abdominal pain

3. Reduction of steatorrhea Rationale: Pancrelipase (Pancrease, Creon) is a pancreatic enzyme used in clients with pancreatitis as a digestive aid. The medication should reduce the amount of fatty stools (steatorrhea). Another intended effect could be improved nutritional status. It is not used to treat abdominal pain or heartburn. Its use could result in weight gain but should not result in weight loss if it is aiding in digestion.

The ambulatory care nurse is preparing to assist the health care provider in performing a liver biopsy on a client. The client is receiving a local anesthetic for the procedure. The nurse should assist the client into which position for this test to be performed? 1. Right lateral side-lying 2. Flat with the head elevated 3. Supine with the right hand under the head 4. Prone with the hands crossed under the head

3. Supine with the right hand under the head Rationale: A client undergoing liver biopsy with use of a local anesthetic will be positioned supine with the client's right hand placed under the head. An alternative position is the left lateral side-lying position. The client also will be asked to remain as still as possible during the test. Pagana Pagana

The nurse is evaluating the plan of care for a client with peptic ulcer disease (PUD) who is experiencing acute pain. The nurse determines that the expected outcomes have not been met if the nursing assessment reveals which result? 1. The client's pain is relieved with histamine-2 receptor antagonists. 2. The client has eliminated any irritating foods from the diet. 3. The client frequently is awakened at 2 am with heartburn. 4. The client reports absence of pain before meals.

3. The client frequently is awakened at 2 am with heartburn. Rationale: Expected outcomes for the client with PUD experiencing pain include elimination of irritating foods from the diet, effectiveness of prescribed medications to reduce pain, self-reporting of absence of pain with medication, and an ability to sleep through the night without pain. The client who continues to be awakened by pain requires further modification of medication therapy, which may include adjustment of timing of histamine-2 receptor antagonist administration or an additional dose of antacid before the time when pain usually awakens the client. IGGY

The nurse is monitoring a client with cirrhosis of the liver for signs of hepatic encephalopathy. Which assessment finding would the nurse note as an early sign of hepatic encephalopathy? 1. Restlessness 2. Complaints of fatigue 3. The presence of asterixis 4. Decreased serum ammonia levels

3. The presence of asterixis Rationale: Asterixis is a flapping tremor of the hand that is an early sign of hepatic encephalopathy. The exact cause of this disorder is not known, but abnormal ammonia metabolism may be implicated. Increased serum ammonia levels are thought to interfere with normal cerebral metabolism. Tremors and drowsiness also would be noted.

A client with gastric hypersecretion is scheduled for surgery. The nurse teaches the client that the procedure will lessen the stomach's production of acid by altering which structure? 1. Portal vein 2. Celiac artery 3. Vagus nerve 4. Pyloric valve

3. Vagus nerve Rationale: Vagotomy is a procedure that can reduce innervation to the stomach, thereby reducing the production of gastric acid. The portal vein drains venous blood from the stomach. The celiac artery brings arterial blood to the stomach. The pyloric valve separates the stomach from the duodenum. The pyloric valve may undergo surgical repair if it becomes stenosed; this procedure is known as pyloroplasty. IGGY

The nurse is explaining an upper gastrointestinal series to a client and provides the client with the preprocedure and postprocedure instructions. The nurse informs the client that after this procedure, the stools can be expected to remain white for what time period? 1. 1 week 2. 6 hours 3. 8 hours 4. 1 to 2 days

4. 1 to 2 days Rationale: It takes at least 12 to 24 hours for a substance to pass through the colon.

A client admitted to the medical nursing unit has a diagnosis of gastroesophageal reflux disease (GERD). Metoclopramide (Reglan) has been prescribed four times a day. The nurse should schedule administration of the medication for which times? 1. Every 8 hours 2. With meals and at bedtime 3. Right after meals and at bedtime 4. 30 minutes before meals and at bedtime

4. 30 minutes before meals and at bedtime Rationale: Metoclopramide is a gastrointestinal stimulant. Administration should be scheduled 30 minutes before meals and at bedtime to allow the medication time to begin working before food intake and digestion.

The nurse is preparing to give a client directions for proper use of aluminum hydroxide tablets. Which instruction by the nurse is appropriate? 1. "Take the tablet at the same time as an antacid." 2. "Swallow the tablet whole with a full glass of water." 3. "Take each dose with a laxative to prevent constipation." 4. "Chew the tablet thoroughly and then drink 8 ounces of water."

4. "Chew the tablet thoroughly and then drink 8 ounces of water." Rationale: Aluminum hydroxide tablets are an antacid and should be chewed thoroughly before swallowing to prevent them from entering the small intestine undissolved. An antacid should not be taken with the medication to prevent additive and interactive effects. Constipation is a side effect of the use of aluminum products, but the client should not take a laxative with each dose. This would promote laxative abuse and should be avoided if less habit-forming means can be used. Kee Hayes

The clinic nurse is providing instructions to a client who is scheduled for a barium enema. What should the nurse instruct the client to do in preparation for this procedure? 1. Liquids are restricted for 24 hours after the test. 2. A clear liquid diet is required for 4 days before the test. 3. Laxatives should not be taken for at least 1 week before the test. 4. A low-fiber diet needs to be maintained for 1 to 3 days before the test.

4. A low-fiber diet needs to be maintained for 1 to 3 days before the test. Rationale: Preparation for a barium enema includes maintaining a low-fiber diet for 1 to 3 days before the test. Clear liquids or water may be allowed 12 to 24 hours before the test. Laxatives and enemas may be prescribed before the test to cleanse the bowel. The client is encouraged to drink liquids after the procedure to facilitate the passage of barium. IGGY & Pagana Pagana

The nurse assists a health care provider in performing a liver biopsy. After the biopsy, the nurse should place the client in which position? 1. Prone 2. Supine 3. A left side-lying position with a small pillow or folded towel under the puncture site 4. A right side-lying position with a small pillow or folded towel under the puncture site

4. A right side-lying position with a small pillow or folded towel under the puncture site Rationale: After a liver biopsy, the client is assisted to assume a right side-lying position with a small pillow or folded towel under the puncture site for 3 hours. This position compresses the liver against the chest wall at the biopsy site.

A client has a large, deep duodenal ulcer diagnosed by endoscopy. Which sign/symptom indicative of a complication should the nurse look for during the client's postprocedure assessment? 1. Bradycardia 2. Nausea and vomiting 3. Numbness in the legs 4. A rigid board-like abdomen

4. A rigid board-like abdomen Rationale: The client with a large, deep duodenal ulcer is at risk for perforation of the ulcer. If this occurs, the client will experience sudden, sharp, intolerable severe pain beginning in the midepigastric area and spreading over the abdomen, which then becomes rigid and board-like. Tachycardia, not bradycardia, may occur as hypovolemic shock develops. Nausea and vomiting may not occur if the pyloric sphincter is intact. Numbness in the legs is not an associated finding. IGGY

The nurse is monitoring a client with a diagnosis of peptic ulcer. Which assessment finding would most likely indicate perforation of the ulcer? 1. Bradycardia 2. Numbness in the legs 3. Nausea and vomiting 4. A rigid, boardlike abdomen

4. A rigid, boardlike abdomen Rationale: Perforation of an ulcer is a surgical emergency and is characterized by sudden, sharp, intolerable severe pain beginning in the mid-epigastric area and spreading over the abdomen, which becomes rigid and boardlike. Nausea and vomiting may occur. Tachycardia may occur as hypovolemic shock develops. Numbness in the legs is not an associated finding. IGGY

A client has undergone esophagogastroduodenoscopy. The nurse should place highest priority on which item as part of the client's care plan? 1. Monitoring the temperature 2. Monitoring complaints of heartburn 3. Giving warm gargles for a sore throat 4. Assessing for the return of the gag reflex

4. Assessing for the return of the gag reflex Rationale: The nurse places highest priority on assessing for return of the gag reflex. This assessment addresses the client's airway. The nurse also monitors the client's vital signs and for a sudden increase in temperature, which could indicate perforation of the gastrointestinal tract. This complication would be accompanied by other signs as well, such as pain. Monitoring for sore throat and heartburn are also important; however, the client's airway is the priority.

Oral neomycin has been prescribed for a client with a diagnosis of portosystemic encephalopathy. The nurse reviews the health care provider's prescription and determines that this medication has been prescribed for which purpose? 1. Prevent infection. 2. Prevent restlessness in the client. 3. Prevent fluid retention and ascites. 4. Destroy normal bacteria found in the bowel.

4. Destroy normal bacteria found in the bowel. Rationale: Neomycin may be prescribed for the client with portosystemic encephalopathy. It is a broad-spectrum antibiotic that destroys normal bacteria found in the bowel, thereby decreasing protein breakdown and ammonia production. Options 1, 2, and 3 are not actions of this medication for this client. IGGY

The nurse is providing care for a client with a Sengstaken-Blakemore tube. The nurse suspects which diagnosis for this client? 1. Gastritis 2. Bowel obstruction 3. Small bowel tumor 4. Esophageal varices

4. Esophageal varices Rationale: A Sengstaken-Blakemore tube may be used in a client with a diagnosis of cirrhosis with ruptured esophageal varices if other treatment measures are unsuccessful. The tube has an esophageal and a gastric balloon. The esophageal balloon exerts pressure on the ruptured esophageal varices and stops the bleeding. The gastric balloon holds the tube in the correct position and prevents migration of the esophageal balloon, which could harm the client. IGGY

A client is diagnosed with a moderate case of acute ulcerative colitis. The nurse doing dietary teaching should give the client examples of foods to eat that represent which therapeutic diet? 1. High fat with milk 2. Low fiber with milk 3. High protein with milk 4. Low fiber without milk

4. Low fiber without milk Rationale: The client with a mild to moderate case of acute ulcerative colitis often is prescribed a diet that is low in fiber and does not include milk. This will help to reduce the frequency of diarrhea for this client. IGGY

A client has a PRN prescription for ondansetron (Zofran). For which condition should the nurse administer this medication to the postoperative client? 1. Paralytic ileus 2. Incisional pain 3. Urinary retention 4. Nausea and vomiting

4. Nausea and vomiting Rationale: Ondansetron is an antiemetic used to treat postoperative nausea and vomiting, as well as nausea and vomiting associated with chemotherapy.

A client has a PRN prescription for trimethobenzamide (Tigan). The nurse should assess the client for which sign or symptom to determine whether the client needs a dose of this medication? 1. Heartburn 2. Constipation 3. Abdominal pain 4. Nausea and vomiting

4. Nausea and vomiting Rationale: Trimethobenzamide is an antiemetic agent used for relief of nausea and vomiting. The medication is not used to treat heartburn, constipation, or abdominal pain. Kee Hayes

A client with a gastric ulcer has a prescription for sucralfate (Carafate), 1 g by mouth four times daily. The nurse should schedule the medication for which times? 1. With meals and at bedtime 2. Every 6 hours around the clock 3. One hour after meals and at bedtime 4. One hour before meals and at bedtime

4. One hour before meals and at bedtime Rationale: Sucralfate is a gastric protectant. The medication should be scheduled for administration 1 hour before meals and at bedtime. The medication is timed to allow it to form a protective coating over the ulcer before food intake stimulates gastric acid production and mechanical irritation.

A nurse is developing a plan of care for a client who will be returning to the nursing unit after a percutaneous transhepatic cholangiogram. The nurse should include which intervention in the postprocedure plan of care? 1. Encourage fluid and food intake. 2. Allow the client bathroom privileges only. 3. Allow the client to sit in a chair for meals. 4. Place a sandbag or other approved device over the insertion site.

4. Place a sandbag or other approved device over the insertion site. Rationale: A percutaneous transhepatic cholangiogram is an x-ray of the biliary duct system that is taken with the use of an iodinated dye instilled via a percutaneous needle inserted through the liver into the intrahepatic ducts. This procedure may be done when a client has jaundice or persistent upper abdominal pain although ultrasound scans and endoscopic retrograde cholangiopancreatography are usually the preferred tests. After this procedure, the nurse monitors the client's vital signs closely for indications of hemorrhage and observes the needle insertion site for bleeding and bile leakage. A sandbag or other pressure device is placed over the insertion site to prevent bleeding. Oral intake is avoided in the immediate postprocedure period in case surgery is necessary to control hemorrhage or bile extravasation, and the client is maintained on bed rest. IGGY

The nurse is assessing a client who is experiencing an acute episode of cholecystitis. Where should the nurse anticipate the location of the pain? 1. Right lower quadrant, radiating to the back 2. Right lower quadrant, radiating to the umbilicus 3. Right upper quadrant, radiating to the left scapula and shoulder 4. Right upper quadrant, radiating to the right scapula and shoulder

4. Right upper quadrant, radiating to the right scapula and shoulder Rationale: During an acute episode of cholecystitis, the client may complain of severe right upper quadrant pain that radiates to the right scapula and shoulder. This is determined by the pattern of dermatomes in the body. The other options are incorrect.

The nurse is performing an assessment on a client with a suspected diagnosis of acute pancreatitis. The nurse will direct the assessment to look for which as a hallmark sign of this disorder? 1. Hypothermia 2. Epigastric pain radiating to the neck area 3. Severe abdominal pain relieved by vomiting 4. Severe abdominal pain that is unrelieved by vomiting

4. Severe abdominal pain that is unrelieved by vomiting Rationale: Nausea and vomiting are common presenting manifestations of acute pancreatitis. A hallmark symptom is severe abdominal pain that is not relieved by vomiting. The vomitus characteristically consists of gastric and duodenal contents. Fever also is a common sign but usually is mild, with temperatures less than 39° C. Epigastric pain radiating to the neck area is not a characteristic symptom. IGGY

A nurse is caring for a group of clients on the surgical nursing unit. The nurse anticipates that the client who underwent which procedure is most likely to have some long-term residual difficulty with absorption of nutrients? 1. Colectomy 2. Appendectomy 3. Ascending colostomy 4. Small bowel resection

4. Small bowel resection Rationale: The small intestine is responsible for the absorption of most nutrients. The client who has undergone removal of a segment of the small bowel is the one who has a decreased area with which to absorb nutrients.

The nurse obtains an admission history for a client with suspected peptic ulcer disease. Which client factor documented by the nurse would increase the risk for peptic ulcer disease? 1. Recently retired from a job 2. Significant other has a gastric ulcer 3. Occasionally drinks one cup of coffee in the morning 4. Takes nonsteroidal anti-inflammatory drugs (NSAIDs) for osteoarthritis

4. Takes nonsteroidal anti-inflammatory drugs (NSAIDs) for osteoarthritis Rationale: Risk factors for peptic ulcer disease include Helicobacter pylori infection, smoking (nicotine), chewing tobacco, corticosteroids, aspirin, nonsteroidal anti-inflammatory drugs (NSAIDs), caffeine, alcohol, and stress. When an NSAID is taken as often as is typical for osteoarthritis, it will cause problems with the stomach. Certain medical conditions such as Crohn's disease, Zollinger-Ellison syndrome, and hepatic and biliary disease also can increase the risk for peptic ulcer disease by changing the amount of gastric and biliary acids produced. Recent retirement should decrease stress levels, rather than increase them. Ulcer disease in a first-degree relative also is associated with increased risk for an ulcer.

Pancreatin (Viokase) is prescribed for a client with postgastrectomy syndrome. Which assessment finding would indicate a therapeutic effect of this medication? 1. The client's appetite improves. 2. The client experiences weight loss. 3. Vitamin B12 deficiency is controlled. 4. The stool is less fatty and decreases in frequency.

4. The stool is less fatty and decreases in frequency. Rationale: Pancreatin aids in the digestion of protein, carbohydrate, and fat in the gastrointestinal tract. It is used to treat steatorrhea associated with postgastrectomy syndrome after bowel resection. The nurse should record the number of stools per day and the stool consistency to monitor the effectiveness of this enzyme therapy. If it is effective, the stools should become less frequent and less fatty.

A home care nurse is visiting a client with a diagnosis of pernicious anemia that developed as a result of gastric surgery. In teaching the client about this condition, the nurse explains that the stomach lining is producing a decreased amount of intrinsic factor, so the client will need which medication? 1. An antacid 2. An antibiotic 3. Vitamin B6 injections 4. Vitamin B12 injections

4. Vitamin B12 injections Rationale: A lack of the intrinsic factor needed to absorb vitamin B12 occurs in pernicious anemia. Vitamin B12 is needed for the maturation of red blood cells. An antacid and antibiotic may be prescribed for certain types of gastric ulcers. Vitamin B6 is not necessarily needed for pernicious anemia and can be taken orally. IGGY


संबंधित स्टडी सेट्स

Chapter 13: Blended Competencies, Clinical Reasoning and Processes of Person-Centered Care

View Set

Adults 1 - Final, Final adult 1 .exm

View Set

6 - Life Insurance Underwriting and Policy Issue

View Set

02 CEH: Footprinting & Reconnaissance

View Set

Organizational Culture: Chapter 16 (OB)

View Set

Chapter 19 Workforce Engagement and Collective Action

View Set

Lecture 11 Opioids: Dependence, Overdoses, and Drug Interactions

View Set